Final Exam: Pathophysiology

¡Supera tus tareas y exámenes ahora con Quizwiz!

The cytoplasm surrounds the nucleus of the cell and contains organelles. Select the organelles found in the cytoplasm. Select all that apply. A. Golgi apparatus B. Endoplasmic reticulum C. Mitochondria D. Lysosomes E. Cytoskeleton

A,B,C,D

A student nurse is taking a test on the endocrine system. From the list of clinical manifestations, she needs to select the ones she would see in hypothyroidism. Which answers should she select? Select all that apply. A. Coarse, brittle hair B. Puffy face with swollen eyelids C. Nervousness with fine muscle tremors D. Heat intolerance E. Weight gain despite loss of appetite

A,B,D

A 51-year-old female client who is 2 days postoperative in a surgical unit of a hospital is at risk of developing atelectasis as a result of being largely immobile. Which teaching point by her nurse is most appropriate? A. "Being in bed increases the risk of fluid accumulating between your lungs and their lining, so it's important for you to change positions often." B. "You should breathe deeply and cough to help your lungs expand as much as possible while you're in bed." C. "I'll prescribe bronchodilator medications that will help open up your airways and allow more oxygen in." D. "Make sure that you stay hydrated and walk as soon as possible to avoid our having to insert a chest tube."

B. "You should breathe deeply and cough to help your lungs expand as much as possible while you're in bed."

A client is admitted to the emergency department with possible internal bleeding after being involved in an automobile accident. What type of isotonic intravenous (IV) solution does the nurse prepare to infuse? A. 0.45% NaCl B. 0.9% NaCl C. 5% Dextrose in water D. 3% NaCl

B. 0.9% NaCl

Which statement best describes morbidity as it relates to disease? A. It is the prediction of how weight of an individual impacts the disease process. B. A negative outcome with disease complications that impact the quality of life. C. A negative outcome with disease complications that causes death. D. It is the prediction of how an individual will proceed through the disease process.

B. A negative outcome with disease complications that impact the quality of life.

The nurse is performing an assessment on a client who states she fell and twisted her right ankle during a softball game. The nurse notes that the right ankle is edematous and will probably need to be x-rayed. What term does the nurse use to describe the changes that accompany this finding? A. A chronic disorder B. A sign C. A syndrome D. A symptom

B. A sign

A patient comes to the clinic concerned about a painless lymph node in the neck. A diagnosis of lymphoma is made. Which of the following would indicate that the lymphoma was Hodgkin lymphoma versus non-Hodgkin lymphoma? A. The location of the enlarged lymph nodes B. The presence of Reed-Sternberg cells C. Spread to the spleen, liver, and bone marrow D. The age of the patient

B. The presence of Reed-Sternberg cells

Following a biopsy, a client has been diagnosed as having a benign neoplastic tumor. Which characteristic most likely applies to his tumor? A. The tumor is poorly approximated and has the potential to break loose. B. The well-differentiated, neoplastic cells are clustered in a single mass. C. The client's tumor has a rapid rate of growth and can induce ischemia. D. The tumor may secrete hormones or cytokines.

B. The well-differentiated, neoplastic cells are clustered in a single mass.

A white blood cell differential shows an increase in the number of monocytes and macrophages in the blood. This typically means that: A. This is a new infection B. This is a chronic infection C. This is a viral infection D. The differential provides no useful information

B. This is a chronic infection

Which of the following areas of the spinal cord contains 12 segments? A. Cervical B. Thoracic C. Sacral D. Lumbar

B. Thoracic

A nurse is speaking to a group of students about the transmission of HIV. Which modes of transmission should be included? Select all that apply. A. Ingesting the saliva of an HIV-positive person B. Skin to skin contact with a person with HIV C. Contact with blood containing HIV D. Contact with the semen of an HIV-positive person E. Transmission from mother to infant through breast milk

C,D,E

The school nurse is preparing a lecture on the human immunodeficiency virus (HIV) for high school students. The nurse includes which information about the transmission of HIV? Select all that apply. A. HIV is transmitted through nonsexual household contact. B. HIV is transmitted through the bite of an insect. C. HIV is transmitted through blood-to-blood contact. D. HIV is transmitted from the mother to the unborn baby. E. HIV is transmitted through sexual contact.

C,D,E

What happens as a cell's workload declines? Select all that apply. A. Insulin levels increase. B. Energy expenditure increases. C. Oxygen consumption decreases. D. Protein synthesis decreases. E. Cell size decreases.

C,D,E

Which description best summarizes the etiology of pressure injuries? A. Dryness of the skin results in the development of pressure injuries for an immobile client. B. Pressure injuries occur due to poor care and are usually preventable, especially in the home setting. C. Unrelieved pressure on the skin causes decreased blood and lymph flow that leads to pressure injury development. D. Pressure injuries occur from skin tears that do not heal properly.

C. Unrelieved pressure on the skin causes decreased blood and lymph flow that leads to pressure injury development

Which of the following situations of altered perfusion could be triggered by chronic obstructive pulmonary disease? A. Impaired cardiac output B. Impaired circulation C. Ventilation-perfusion mismatching D. Excessive cardiac demand

C. Ventilation-perfusion mismatching

A patient is taking an anti-inflammatory drug for rheumatoid arthritis. What is the most likely action for this drug? A. Blocks the chemical mediators of inflammation B. Enhances the body's immune system C. Increases blood flow to the tissues D. Decreases scar formation

A. Blocks the chemical mediators of inflammation

A patient wants to know what has caused his illness. This information is termed the: A. Etiology B. Pathogenesis C. Epidemiology D. Nosocomia

A. Etiology

The nurse recognizes that both grading and staging are methods for classifying cancer and selecting a treatment plan. What is clinical staging used to determine? A. Extent and spread of the disease B. Number of mitoses C. Histologic characteristics D. Level of differentiation

A. Extent and spread of the disease

The underlying causative problem in Parkinsonism is: A. Failure of dopamine release B. Viral infection C. Autoimmune disorder D. Genetic defect

A. Failure of dopamine release

Immunodeficiency is the result of: A. Failure of host defense mechanisms B. Hypersensitive immune response C. Inappropriate immune response to self D. Immune response stimulated by antigens from other individuals

A. Failure of host defense mechanisms

Which of the following occurs with elevated levels of cortisol? A. Fatty acids are mobilized B. Glucose levels are suppressed C. Plasma protein levels increase D. Inflammation increases

A. Fatty acids are mobilized

Respiratory alkalosis can be caused by a respiratory rate in excess of that which maintains normal plasma PCO2 levels. What is a common cause of respiratory alkalosis? A. Hyperventilation B. Kussmaul breathing C. Cluster breathing D. Hypoventilation

A. Hyperventilation

The nurse cares for a client with Crohn disease. Which stool pattern will the client report? A. Intermittent nonbloody diarrhea B. Hard and pebble-like stool C. Alternating constipation and diarrhea D. Fatty semi-formed stool

A. Intermittent non bloody diarrhea

The nurse is assessing a client with diabetes and notes an area on the client's right foot as inflamed, necrotic, and eroded. The client states he accidentally slammed his foot in a door 2 weeks ago. The nurse would document this finding as a(n): A. Ulceration B. Fungus C. Pustule D. Abscess

A. Ulceration

Which of the following conditions represents pathologic responses caused by immunologic memory? A. Common cold B. Anaphylaxis C. Shingles D. Strep throat

B. Anaphylaxis

A nurse is caring for a client following surgery. The central venous pressure (CVP) monitor indicates low pressures. Which action is a priority for the nurse to take? A. Elevate the head of the client's bed to 45 degrees. B. Increase the IV fluid infusion per protocol. C. Document the CVP and continue to monitor. D. Administer oxygen 2L per nasal cannula.

B. Increase the IV fluid infusion per protocol

At what point in the cardiac cycle do you measure preload? A. Just after systole B. Just before systole C. During systole D. It is not possible to measure preload

B. Just before systole

Which cellular mediator is involved in the development of type I hypersensitivity reactions? A. Plasma cells B. Mast cells C. Arachidonic acid D. Monocytes

B. Mast cells

How would you know you have hypoxemia? A. Radiograph B. Measure the partial pressure of oxygen in blood C. Measure the partial pressure of carbon dioxide in blood D. All of these detect hypoxemia

B. Measure the partial pressure of oxygen in blood

Water movement from the side of the membrane having a lesser number of particles and greater concentration of water to the side having a greater number of particles and lesser concentration of water is termed: A. Active transport B. Osmosis C. Filtration D. Diffusion

B. Osmosis

Which cell type has the most potential for regeneration after injury? A. Astrocyte B. Peripheral axon C. Glial cell D. Oligodendrocytes

B. Peripheral axon

Which type of lung cancer is associated with the best prognosis in most clients? A. Large cell carcinoma B. Squamous cell carcinoma C. Small cell lung cancer D. Adenocarcinoma

B. Squamous cell carcinoma

At your health screening, you describe the following: achiness, lethargy, and vague abdominal discomfort. These are categorized as: A. Local manifestations B. Systemic manifestations C. Signs D. Symptoms

B. Systemic manifestations

The nurse is teaching a client infected with the flu about viruses. Which explanation would the nurse use to describe a viral infection? A. It is caused by a hypersensitivity reaction to a biologic agent. B. An irritant causes an inflammatory response. C. A parasitic relationship between an organism and host is present. D. It is caused by a defect in the immune system.

C. A parasitic relationship between organism and host is present

The most reliable method for measuring body water or fluid volume increase is by assessing: A. Intake and output B. Serum sodium levels C. Body weight change D. Tissue turgor

C. Body weight change

A major difference between the acute and chronic inflammatory response is that in chronic inflammation: A. Inflammatory mediators are released B. Neutrophils are much more prominent C. Granulomas form around certain invaders D. Granulation tissue is present

C. Granulomas form around certain invaders

In 2014, an outbreak of Ebola virus disease began in West Africa in which there was a sharp increase in the number of people being diagnosed with this disease. At the same time, a very high proportion of those who were diagnosed died from the disease. The epidemiologic characteristics of this disease include: A. Increased morbidity and high prevalence. B. High prevalence and increased incidence. C. Increased incidence and high mortality. D. High morbidity and mortality.

C. Increased incidence and high mortality

When caring for a client with hyperkalemia, the nurse prioritizes assessment of which body system? A. Hepatic B. Cerebrovascular C. Pulmonary D. Cardiovascular

D. Cardiovascular

Which of the following is characteristic of benign neoplasms? A. Highly undifferentiated B. Invasive C. Destructive D. Cell overproliferation

D. Cell overproliferation

A client tells the nurse that the primary care provider prescribed an antibiotic. The client was instructed to take the antibiotic three times a day for 10 days. After the 4th day, the client was feeling much better and decided to stop taking the medication. What complication could result from stopping the antibiotic early? A. Septic shock B. Bacteremia C. Septicemia D. Chronic infection

D. Chronic infection

A client has been diagnosed with metabolic acidosis. What assessment finding does the nurse expect? A. Increased PCO2 above 45 mm/Hg (5.99 kPa) B. Increased pH above 7.45 C. Decreased PCO2 below 35 mm/Hg (4.66 kPa) D. Decreased pH below 7.35

D. Decreased pH below 7. 35

What is the major problem in cystic fibrosis? A. Formation of cysts in fibrotic tissues B. Pancreatitis C. Lung injury D. Electrolyte and water transport

D. Electrolyte and water transport

Which type of hormone is not produced in the adrenal cortex? A. Mineralocorticoids B. Glucocorticoids C. Adrenal sex hormones D. Neurotransmitters

D. Neurotransmitters

The condition of a client with metabolic acidosis from an intestinal fistula is not improving. The pulse is 125 beats/min and the BP 84/56 mm Hg. ABG values are: pH 7.1, HCO3- 18 mEq/L (18 mmol/L), PCO2 57 mm Hg (7.58 kPa). What IV medication should the nurse expect to provide next? A. Antibiotics B. Epinephrine C. Potassium chloride D. Sodium bicarbonate

D. Sodium bicarbonate

A nurse is teaching a group of clients about causes/risk factors for cancer, which include which factors? Select all that apply. A. Genetics B. Age C. Environment D. Heredity E. Financial status

A,B,C,D

Hyperthyroidism that is inadequately treated can cause a life-threatening condition known as a thyroid storm. What are the manifestations of a thyroid storm? Select all that apply. A. Delirium B. Tachycardia C. Very low fever D. Very high fever E. Bradycardia

A,B,D

The family of a multiple sclerosis client asks, "What psychological manifestations may we expect to see in our mother?" The health care provider informs them to expect which of the following? Select all that apply. A. Forgetfulness B. Depression C. Delirium D. Hallucinations E. Inattentiveness

A,B,E

The clinic nurse suspects the client is having a genetically determined hypersensitivity to common environmental allergens since the client is experiencing which clinical manifestations? Select all that apply. A. Runny nose B. Diarrhea C. Hives (urticaria) D. Wheezes E. Topical pustules

A,C,D

The nurse is monitoring trends in the client's arterial blood gases and recognizes that changes in ventilation will result from which of the following? Select all that apply. A. Carbon dioxide B. Anion gap C. pH D. Oxygen E. Cerebrospinal fluid (CSF)

A,C,D

When comparing the endocrine and nervous system functions, the nurse knows that the endocrine system: Select all that apply. A. Takes longer to respond to innervations but has prolonged actions when they arrive. B. Responds to neurotransmitter molecules within milliseconds. releases hormones into the blood that is transported throughout the body. C. Glands are widely scattered throughout the body. D. Sends signals to neurons over a short distance to muscles.

A,C,D

Which clinical manifestations following thyroidectomy would alert the nurse that the client is going into a life-threatening thyroid storm? Select all that apply. A. Telemetry showing heart rate of 184 B. Bruising on knees and feet C. Extremely agitated D. Temperature of 104.2°F (40.1°C) E. Unable to close eyelids completely together

A,C,D

While studying cancer, nursing students learn about the process by which cancer-causing agents cause normal cells to become cancerous. This process is a multi-step mechanism that can be divided into three stages, which include which of the following? Select all that apply. A. Initiation B. Evaluation C. Promotion D. Progression E. Termination

A,C,D

A family member comes rushing out of a client's room telling the nurse that the loved one can't breathe. The nurse has just left the room after hanging IV penicillin. Which clinical manifestations lead the nurse to suspect the client is experiencing anaphylactic shock? Select all that apply. A. Wheezing sound on inspiration B. Incontinent of urine C. Hives over entire body D. Swelling around the lips and eyes E. Severe bronchospasm

A,C,D,E

A client has been diagnosed with dysfunction of the anterior pituitary gland. The nurse is aware that which hormones will likely be affected? Select all that apply. A. Luteinizing hormone (LH) B. Norepinephrine C. Adrenocorticotropic hormone (ACTH) D. Growth hormone (GH) E. Antidiuretic hormone (ADH) F. Thyroid-stimulating hormone (TSH)

A,C,D,F

A nurse doing a presentation on cancer to a local community group identifies which factors to be high-risk and associated with cancer development? Select all that apply. A. Immunologic mechanisms B. Low weight C. Heredity D. Hormonal factors E. Number of siblings F. Environmental agents

A,C,D,F

A client with a history of heart failure and COPD (caused by 60 pack/year smoking) presents to the clinic reporting difficulty breathing. Auscultation of breath sounds reveal absent/diminished breath sounds in the right lower lobe. Which other manifestations lead the health care provider to suspect the client may have developed atelectasis? Select all that apply. A. Using accessory muscles to help him breathe. B. Copious amounts of thick, green sputum. C. "Seems like I'm not making much water (decreased urine production)." D. "Having a hard time catching my breath." E. Respiratory rate—32; pulse rate—122 beats/min.

A,D,E

A nurse is counseling a client about risk factors for yeast infections. Which should the nurse list as a risk factor for an overgrowth of Candida albicans? Select all that apply. A. Impaired immune system B. Core body temperature of 37°C (98.6°F) C. Recent exposure to a person with athlete's foot D. Decrease in amount of bacterial flora E. Antibiotic therapy

A,D,E

The nurse is caring for a client who has sustained injury to the adrenal medulla as a result of a penetrating wound due to a trauma. Which hormones does the nurse expect to be altered as a result of the client's injury? Select all that apply. A. Norepinephrine B. Estrogen C. Mineralocorticoids D. Glucocorticoids E. Epinephrine

A,E

A client with heart failure asks, "Why am I taking a 'water pill' when it's my heart that is having a problem?" While educating the client about the Frank-Starling mechanism, which explanation is most appropriate to share? A. "Since your heart is not pumping efficiently, the kidneys are getting less blood flow; therefore, the kidneys are holding on to sodium and water." B. "Since your heart function is impaired, the lungs are not able to oxygenate the blood and your kidneys are wearing out." C. "You must be drinking way too many liquids. Your kidneys cannot filter all that you are drinking during the day." D. "Your heart muscle is overstretched, so it's not able to pump all the blood out. The prescribed 'water pills' help by decreasing your weight."

A. "Since your heart is not pumping efficiently, the kidneys are getting less blood flow; therefore, the kidneys are holding on to sodium and water."

A nurse is providing care for several clients on an acute medicine unit. Which client should the nurse recognize as being at the highest risk for metabolic alkalosis? A. A client on continuous nasogastric suction and whose hypertension is being treated with diuretics B. A client with acquired immunodeficiency syndrome (AIDS) who has developed tuberculosis and is receiving antibiotics C. A postoperative client who developed sepsis after the dehiscence of an abdominal wound D. A client in alcohol withdrawal who is being treated with intravenous anticonvulsants

A. A client on continuous nasogastric suction and whose hypertension is being treated with diuretics

A member of the health care team is researching the etiology and pathogenesis of a number of clients who are under her care in a hospital context. Which client situation best characterizes pathogenesis rather than etiology? A. A client who has increasing serum ammonia levels due to liver cirrhosis B. A client who has been exposed to the mycobacterium tuberculosis bacterium C. A client who was admitted with the effects of methyl alcohol poisoning D. A client with multiple skeletal injuries secondary to a motor vehicle accident

A. A client who has increasing serum ammonia levels due to liver cirrhosis

Dyspnea can best be described by which of the following? A. A subjective sensation felt when experiencing difficulty breathing B. Abnormally fast and deep breathing that results in retaining carbon dioxide C. Very rapid respirations D. Breathing that is rhythmic waxing and waning of the depth of respiration

A. A subjective sensation felt when experiencing difficulty breathing

The nurse is assessing a male client and finds abnormally large hands and feet, a bulbous nose, and a broad face with a protruding jaw. Based on these findings, which endocrine abnormality is most likely the cause for these physical changes? A. Acromegaly B. Cushing syndrome C. Myxedema D. Hyperthyroidism

A. Acromegaly

The cell's typical response to a decrease in trophic signal is: A. Atrophy B. Hypertrophy C. Hyperplasia D. Phagocytosis

A. Atrophy

Which of the following scenarios is most likely in the CSF of a patient with bacterial meningitis? A. CSF has high neutrophil count and high protein count B. CSF has high neutrophil count and low protein count C. CSF has high glucose level and high red blood cell count D. CSF has high lymphocyte count and low red blood cell count

A. CSF has high neutrophil count and high protein count

The nurse in the oncology unit has just admitted a client with metastatic cancer. The client asks how cancer moves from one place to another in the body. What would the nurse answer? A. Cancer cells enter the body's lymph system and thereby spread to other parts of the body. B. Cancer cells are moved from one place in the body to another by transporter cells. C. The cancer cells are not able to float around the original tumor in body fluids. D. Cancer cells replicate and form a chain that spreads from the original tumor site to the site of the metastatic lesion.

A. Cancer cells enter the body's lymph system and thereby spread to other parts of the body

Which of the following genes, when mutated, is NOT implicated in the development of neoplasms? A. Cancinogenes B. Tumor suppressor genes C. Oncogenes D. Mutator genes

A. Cancinogenes

The nurse is assessing a client with abnormal blood gas values. Which of these interpretations does the nurse make regarding the client's physiologic status? A. The client has abnormalities in his buffer system. B. The client has abnormalities in his cardiovascular system. C. The client has alterations in vital signs. D. The client has a decreased hemoglobin.

A. Client has abnormalities in his buffer system

Which of the following clinical manifestations are related to hypoxemia? A. Cyanosis B. Cough C. Chest pain D. Hemoptysis

A. Cyanosis

A nurse is caring for a client with severe liver failure. Knowing that the client's condition has resulted in impaired synthesis of albumin, the nurse understands that the client's generalized edema is related to which of the following? A. Decreased colloidal osmotic pressure B. Increased capillary permeability C. Increased hydrostatic pressure D. Obstruction of lymphatic flow

A. Decreased colloidal osmotic pressure

Cells develop into tissues with specialized structure and function through the process of: A. Differentiation B. Proliferation C. Endocytosis D. Exocytosis

A. Differentiation

Which one of the following neurotransmitters is deficient in Parkinson disease? A. Dopamine B. Serotonin C. Norepinephrine D. Acetylcholine

A. Dopamine

Infections cause a local inflammatory response at the site of infection, which leads to specific clinical manifestations. In the case of pyelonephritis, what would be a likely clinical manifestation? A. Dysuria B. Hyperglycemia C. Tachycardia D. Pruritis

A. Dysuria

Which statement will the nurse include when teaching a client with Crohn disease about dietary restrictions during an exacerbation? A. "Eat foods low in fat and fiber." B. "Avoid dairy foods and eggs." C. "Avoid seeds and whole grains." D. "Eat foods high in fat and calories."

A. Eat food low in fat and fiber

A client with a suspected diagnosis of primary hypothyroidism would most likely demonstrate which serum laboratory value? A. Elevated thyroid-stimulating hormone (TSH) and decreased thyroxine (T4) B. Decreased thyroid-stimulating hormone (TSH) related to poor thyroid function C. Elevated thyroid-stimulating hormone (TSH), thyroxine (T4), and T3 D. Decreased thyroid-stimulating hormone (TSH) and thyroxine (T4), and low T3

A. Elevated thyroid-stimulating hormone (TSH) and decreased thyroxine (T4)

Health officials determine that the annual influenza vaccine is effective for the strain of flu, which appears stable and predictable in its incidence and prevalence. How will the nurse describe this strain of influenza? A. Endemic B. Systemic C. Epidemic D. Pandemic

A. Endemic

A health care provider has completed an assessment on a client diagnosed with cirrhosis. The client asks, "What, if any, serious complications are associated with cirrhosis?" Which response is the most accurate for the provider to relay to the client? A. Esophageal varices B. Dehydration C. Biliary colic D. Duodenal ulcers

A. Esophageal varices

A client is admitted after losing 44 lb (20 kg) over the past 3 months, largely due to frequent vomiting. What intervention should the nurse anticipate in the treatment of the client's resulting acid-base imbalance? A. Fluid replacement with an intravenous solution containing electrolytes as prescribed B. Supplementary oxygen using a non-rebreather mask C. Administration of intravenous sodium bicarbonate as prescribed D. Mechanical ventilation and administration of supplementary oxygen

A. Fluid replacement with an intravenous solution containing electrolytes as prescribed

Which lab result strongly suggests an immunologic response in the client with possible rheumatic heart disease? A. Group A (β-hemolytic) streptococcal antibodies B. High C-reactive protein levels C. Elevated white blood cell count D. Elevated erythrocyte sedimentation rate (ESR)

A. Group A (β-hemolytic) streptococcal antibodies

Which of the following hormones is secreted from the anterior pituitary? A. Growth hormone B. Somatostatin C. Growth hormone releasing hormone D. Insulin-like growth factor 1

A. Growth hormone

A health care provider suspects a client may have developed pancreatitis. Which laboratory value will confirm this diagnosis? A. High serum amylase and lipase B. Altered alkaline phosphatase and red blood cell count C. Change in platelet count and prothrombin level C. Chymotrypsin level and fibrinogen level

A. High serum amylase and lipase

Which mechanism increases peripheral vascular resistance and contributes to the development of hypertension? A. Impaired sodium excretion by the kidneys B. Parasympathetic nervous system overstimulation C. Reduced renin-angiotensin-aldosterone secretion D. None of these

A. Impaired sodium excretion by the kidneys

You are expecting your first child and are told that the child has a 1 in 800 chance of being born with a congenital anomaly. This statistic refers to the: A. Incidence B. Prevalence C. Epidemic D. Diagnosis

A. Incidence

Which sequence accurately describes the stages of a disease? A. Incubation, prodromal, acute, convalescent, and resolution. B. Incubation, prodromal, current, recovery, and resolution C. Prodromal, subacute, acute, postdromal, and resolution D. Subacute, prodromal, acute, postacute, and convalescent

A. Incubation, prodromal, acute, convalescent and resolution

A school-age child with a history of asthma has brought a note home from school stating there has been one case of meningitis (Neisseria meningitidis) in the school. Since the mother is a nurse, she is very concerned since she knows the portal of entry of this pathogen is: A. Inhalation via the respiratory tract such as through breathing or yawning. B. Ingestion such as when children share their drink with their friends. C. Through a cut or abrasion that may occur on the playground. D. Direct contact with a contaminated object such as a pencil.

A. Inhalation via the respiratory tract such as through breathing or yawning

Which of the following pathways best describes the pathophysiology of acute respiratory distress syndrome? A. Injury-inflammation-pulmonary edema-alveolar collapse-hypoxemia-fibrosis B. Infection-edema-fibrosis-hypoxemia-alveolar collapse-pulmonary edema C. Genetic mutation-sodium transport impaired-mucous stasis-infection-hypoxemia D. Trigger-inflammation-airway constriction-alveolar collapse-hypoxemia-fibrosis

A. Injury-inflammation-pulmonary edema-alveolar collapse-hypoxemia-fibrosis

During flu season, you get exposed to the influenza virus. Which component of your immune system will be the first to respond to this foreign pathogen? A. Innate B. Adaptiive C. Humoral D. T-cell mediated

A. Innate

Which one of the following conditions will most likely result in edema? A. Intravascular filtration pressure greater than interstitial filtration pressure B. Intravascular reabsorption greater than interstitial reabsorption C. Intravascular oncotic pressure greater than interstitial oncotic pressure D. Increased lymphatic reabsorption

A. Intravascular filtration pressure greater than interstitial filtration pressure

The student attends a health fair and has his serum cholesterol checked. He has a high lipoprotein level (LDL). He understands which of the following about LDL cholesterol? A. It is believed to play an active role in the pathogenesis of the atherosclerotic lesion. B. It transports cholesterol away from cells to the liver for excretion. C. It is associated with a low intake of saturated fats. D. It has a low cholesterol content.

A. It is believed to play an active role in the pathogenesis of the atherosclerotic lesion.

The nurse is reviewing laboratory data for the client with an anion gap of 17. The nurse recognizes which condition is associated with an increased anion gap mEq/L (mmol/L)? A. Lactic acidosis B. Multiple myeloma C. Hyperkalemia D. Hypermagnesemia

A. Lactic acidosis

The release of hormones from glands is most often controlled by: A. Negative feedback mechanisms B. Nephrogenic mechanisms C. Ectopic hormone production D. Active transport

A. Negative feedback mechanisms

A 60-year-old woman has been recently diagnosed with multiple sclerosis, a disease in which the oligodendrocytes of the client's central nervous system (CNS) are progressively destroyed. Which physiologic process within the neurologic system is most likely to be affected by this disease process? A. Nerve conduction B. Production of cerebrospinal fluid C. Oxygen metabolism D. Neurotransmitter synthesis

A. Nerve conduction

A school nurse is teaching a class on immunity. Which statement contains an accurate explanation about cellular defenses? A. Neutrophils engulf invading organisms where lysosomes break them down. B. Cells release enzymes into the extracellular fluid and this degrades invading organisms. C. Ion channels allow rapid polarity changes, which inactivate invading organisms. D. Active transport draws invading organisms through cell channels to be destroyed.

A. Neutrophils engulf invading organisms where lysosomes break them down

When a client with a history of asthma takes a walk outside on a windy day with high pollen counts, she may experience an asthma attack, resulting in an increase in respiration rate and wheezing. The body's response is likely related to which pathophysiologic principle? A. Parasympathetic nervous system stimulation resulting in airway constriction B. Release of catecholamines causing blood vessel constriction C. Influx of macrophages to wall of the pollen, thereby stopping the attack D. Inhibition of glandular secretions, which causes build up of mucus in the lungs

A. Parasympathetic nervous system stimulation resulting in airway constriction

Which of the following is true regarding cytotoxic T lymphocytes? A. Recognize the MHC class 1-antigen complex B. Recognize the MHC class 2-antigen complex C. Carry the CD4 marker D. Present antigen to B lymphocytes for antibody production

A. Recognize the MHC class 1-antigen complex

A wound is 6 cm × 6 cm × 4 cm. A wound with these dimensions needs to heal through: A. Secondary intention B. Primary intention C. Tertiary intention D. Scar tissue formation

A. Secondary intention

A 20-year-old college student being treated for a kidney infection developed a temperature of 104ºF (40°C) in spite of treatment with antibiotics. Her pulse was high, her blood pressure was low, and her skin was hot, dry, and flushed. The nurse knows that this client most likely is experiencing which type of shock? A. Septic B. Cardiogenic C. Neurogenic D. Anaphylactic

A. Septic

Primary adrenal insufficiency is manifested by: A. Serum sodium level of 120 mEq/L (120 mmol/L) (low) and blood glucose level of 48 mg/dL (2.66 mmol/L) (low) B. Potassium level of 2.8 mEq/L (2.8 mmol/L) and weight gain of 3 pounds overnight C. Truncal obesity and 3+ pitting edema in lower legs D. Hypopigmentation over neck and BP greater than 150/90

A. Serum sodium level of 120 mEq/L and blood glucose level of 48 mg/dL

Your neighbor tells you that she was in the hospital and they told her she had a bundle branch block. What does this mean? A. She cannot conduct electrical impulses to stimulate the heart ventricle to contract B. She has had a cardiac arrest C. She is experiencing obstructed P wave depolarization of the atria D. She has an excessive calcium influx causing prolonged myocardial contraction

A. She cannot conduct electrical impulses too stimulate the heart ventricle to contract

You are a public health official and want to make the greatest difference in reducing cancer deaths across the globe. Which intervention would have the greatest impact? A. Smoking cessation programs B. Routine genetic testing C. Distribute sunscreen to all adults and children D. Eliminate unnecessary uses of radiation

A. Smoking cessation programs

An anion gap of 18 can be calculated by which of the following scenarios? A. Sodium 146, chloride 102, bicarbonate 26 mEq/L B. Sodium 140, chloride 102, bicarbonate 26 mEq/L C. Sodium 136, chloride 122, bicarbonate 30 mEq/L D. Sodium 148, chloride 100, bicarbonate 28 mEq/L

A. Sodium 146, Chloride 102, Bicarbonate 26 mEq/L

A 6-month-old infant has been hospitalized with acute bronchiolitis. Which treatment should be prioritized in the infant's care? A. Supplementary oxygen therapy B. Tracheotomy C. Transfusion of fresh frozen plasma D. Intravenous antibiotics

A. Supplementary oxygen therapy

Neurogenic shock is due to altered transmission in which conduction system? A. Sympathetic B. Parasympathetic C. Somatic D. Peripheral

A. Sympathetic

In addition to being the site of gas exchange, the lungs also perform which function? A. Synthesize pulmonary surfactant B. Activate vasoactive substances C. Use water vapor pressure to inflate alveoli D. Empty extra blood volume into the left heart

A. Synthesize pulmonary surfactant

A client with pulmonary hypertension has hypertrophy of the heart's right ventricle. Which explanation is appropriate to tell the client? A. "The cells enlarge in your heart from the increased workload." B. "Hypertrophy is a result of necrotic changes from hypoxia." C. "As cells in your heart die, they get smaller." D. "As your heart works harder, the number of cells increases."

A. The cells enlarge in your heart from the increased workload

The nurse knows that the primary long-term regulation of blood pressure is exerted by which body system? A. The kidneys B. Neural mechanisms C. Hormonal activity D. Humoral influence

A. The kidneys

A client presents to the physician's office with a chronic cough, shortness of breath, and wheezing that has gotten progressively worse, with recent episodes of hemoptysis. Diagnostic tests reveal a lung mass. Based on these symptoms, the nurse anticipates which of the following has occurred? A. The lesion has eroded blood vessels in the lungs. B. The lesion has invaded the mediastinum. C. The client has developed a pleural effusion. D. The client has developed superior vena cava syndrome.

A. The lesions have eroded blood vessels in the lungs

Release of hormones from the anterior pituitary differs from those released by the posterior pituitary. What is the major way in which these are different? A. The posterior pituitary is much less complicated B. The anterior pituitary releases only two hormones C. The posterior pituitary follows the positive feedback loop D. The anterior pituitary is controlled by the hypothalamus

A. The posterior pituitary is much less complicated

Depolarization involves: A. The rapid movement of sodium into the cell B. The movement of potassium ions out of the cell C. Movement of potassium ions into the cell D. The absence of electrical activity

A. The rapid movement of sodium into the cell

With acute respiratory distress syndrome (ARDS), a client progressively increases his work of breathing. The physiologic principle behind this respiratory distress is related to: A. The stiffening of the lung, making it more difficult to inflate. B. The elevation of pulmonary venous pressure. C. Increases in left atrial pressure causing thickening of the lining of the pulmonary arteries. D. Structural abnormalities of pulmonary vessels with proliferation of the vessel intima.

A. The stiffening of the lung, making it more difficult to inflate

Select the response that correctly identifies B lymphocytes. A. They are the only cells capable of producing antibodies. B. They have a subset of T cells called helper T cells. C. They are responsible for cell-mediated immunity. D. They do not differentiate into plasma cells, which produce antibody.

A. They are the only cells capable of producing antibodies

A client with a suspected MI is brought to the emergency department by ambulance. The nurse caring for this client would expect to receive an order for which laboratory test to confirm a diagnosis of MI? A. Troponin level B. Calcium level C. Creatine kinase marker D. Complete blood components

A. Troponin level

A hypersensitivity reaction resulting from a yellow jacket sting is an example of: A. Type 1, immediate hypersensitivity reaction B. Type 2, antibody-mediated reaction C. Type 3, immune complex reaction D. Type 4, cell-mediated reaction

A. Type 1, immediate hypersensitivity reaction

A 5-year-old girl's diagnosis of bone cancer required an aggressive treatment regimen. Which consideration forms the most significant threat to her future health? A. Unwanted effects of chemotherapy and radiation therapy B. Resistance to chemotherapy and radiation if required later in life C. Risk for recurrence of the primary neoplasm after puberty D. Retention of chemotherapeutic drugs in the healthy bone matrix

A. Unwanted effects of the chemotherapy and radiation therapy

The vertebral column provides protection of which parts of the body? Select all that apply. A. Muscles B. Spinal nerves C. Supporting structures D. Spinal cord E. Tendons

B,C,D

Which are mechanisms by which pathogens cause disease in humans? Select all that apply. A. Attack of the pathogen by the host cell's immune cells B. Interference with the host cell's metabolic function C. Direct destruction of the host cell D. Exposure of the host cell to toxins

B,C,D

Select the assessment data that place a client most at risk for the development of an opportunistic infection. A client who: Select all that apply. A. Has normal nutrition and body weight. B. Has a compromised immune system. C. Is currently receiving chemotherapy. D. Has a diagnosis of malnutrition. E. Just completed 6 weeks of radiation therapy.

B,C,D,E,

A client pulled from a burning house is transported to the emergency department of a local hospital and found to have severe burns. The nurse is aware that the client is at risk for: Select all that apply. A. Fluid gain B. Airway problems C. Pain D. Hemodynamic instability E. Infection F. Emotional stress

B,C,D,E,F

A client experienced the onset of chest pain and collapses and dies suddenly. The family asks, "What caused him to die so suddenly?" Which response by the health care provider is mostappropriate? A. Hypertrophic cardiomyopathy B. Acute ventricular dysrhythmia C. High troponin levels D. Acute myocarditis

B. Acute ventricular dysrhythmia

A client with a long history of alcohol abuse has been admitted to the emergency department after several of days of heavy drinking. The nurse can best promote the restoration of the client's acid-base balance by: A. Administering intravenous magnesium sulfate as prescribed. B. Administering intravenous sodium bicarbonate as prescribed. C. Positioning the client in an upright position when in bed and mobilizing the client frequently. D. Encouraging pursed-lip breathing and deep breathing and coughing exercises.

B. Administering intravenous sodium bicarbonate as prescribed

Neurons that carry sensory information to distant parts of the brain and spinal cord are called: A. Efferent neurons B. Afferent neurons C. Interneurons D. Extraneurons

B. Afferent neurons

Which agent is considered to be high risk in a bioterrorism event? A. M. tuberculosis B. Bacillus anthracis C. Hantavirus D. E. coli

B. Bacillus anthracis

A diagnosis of acquired immunodeficiency syndrome (AIDS) is identified when the CD4+ T cell count reaches which level? A. 800 to 1000 cells/μL B. Below 200 cells/μL C. 500 to 800 cells/μL D. 200 to 499 cells/μL

B. Below 200 cells

A client is diagnosed with an adenoma. The nursing student identifies this as a: A. Malignant tumor of glandular tissue. B. Benign tumor of glandular epithelial tissue. C. Malignant tumor of bone tissue. D. Benign tumor of bone tissue.

B. Benign tumor of glandular epithelial tissue

A client who is diagnosed with breast cancer asks the nurse if cancer cells ever die. Which statement is the nurse's best response? A. "You need to ask your doctor about this." B. "Cancer cells differ from normal cells by being immortal and have an unlimited life span." C. "It really does not matter as it is more important to work on killing them." D. "Knowing this is not that important and you should not worry about that."

B. Cancer cells differ from normal cells by being immoral and have an unlimited life span

Ascites is the primary mechanism of body fluid imbalance in which of the following conditions? A. Salt-losing tubulopathy B. Cirrhosis C. AIDS D. Isonatremic dehydration

B. Cirrhosis

Crohn disease is recognized by sharply demarcated, granulomatous lesions that are surrounded by normal-appearing mucosal tissue. The nurse recognizes these lesions to be defined by which description? A. Pyramidal B. Cobblestone C. Mosaic D. Triangular

B. Cobblestone

Why is Crohn disease more likely to cause intestinal obstruction than ulcerative colitis? A. Crohn disease is located in the small intestine B. Crohn disease causes granulomas to form in the submucosal layer C. Crohn disease causes abdominal pain and watery diarrhea D. Crohn disease is exacerbated by certain foods, such as spicy foods

B. Crohn disease causes granulomas to form in the submucosal layer

The nurse is developing a plan of care for a client with heart failure. The most important information for the nurse to consider would be: A. Increased ejection fraction B. Decreased cardiac output C. Increased renal blood flow D. Decreased retention of sodium

B. Decreased cardiac output

Depth of injury is important to determine with burns. You are in the sun too long without sunscreen and develop redness and blistering on your face, chest, and back. What depth of burn did you experience? A. Superficial partial-thickness burn B. Deep partial-thickness burn C. Full-thickness burn D. Dermal thickness burn

B. Deep partial-thickness burn

The nurse is explaining to the parents of a 23-week premature infant the reason their baby needs to be on mechanical ventilation. The education is successful when the parents state that they understand their baby was born before the type II alveolar cells could mature, which has caused a: A. Deficiency of type I alveolar cells. B. Deficiency of surfactant. C. Deficiency of alveoli. D. Deficiency of conducting airways.

B. Deficiency of surfactant

A client who has been taking acetaminophen 1000 mg every 4 hours presents to the Urgent Care Center with increased abdominal pain, elevated ALT, AST, and bilirubin levels. The nurse suspects the client is experiencing: A. Chronic active hepatitis B. Direct hepatotoxic reaction C. Cholestatic reaction D. Idiosyncratic reaction

B. Direct hepatotoxic reaction

Which of the following terms describes cells that are enlarged, with darkened nuclei and abnormal chromatin A. Hyperplastic B. Dysplastic C. Metaplastic D. Aplastic

B. Dysplastic

What sign/symptom might indicate that you are experiencing a paraneoplastic syndrome? A. Bone pain B. Edema C. Cough D. Hematuria

B. Edema

Which of the following is not a major role of hormones? A. Growth stimulation B. Erythrocyte synthesis C. Fluid balance and regulation D. Metabolic rate regulation

B. Erythrocyte synthesis

A specific type of gram-negative bacteria contains endotoxin in the bacterial cell envelope. What is the likely clinical manifestation if these bacteria become pathogenic? A. Leukopenia B. Fever C. Constipation D. Vomiting

B. Fever

Inflammation can be either local or systemic. What are the most prominent systemic manifestations of inflammation? A. Fever, leukocytosis or leukopenia, and the transition phase response B. Fever, leukocytosis or leukopenia, and the acute phase response C. Widening pulse pressure, thrombocytopenia, and the recovery phase response D. Widening pulse pressure, thrombocytopenia, and the latent phase response

B. Fever, leukocytosis or leukopenia, and the acute phase response

It is a hot summer day. Your neighbor stops at your house after jogging 5 miles. She is sweating and tells you she feels dizzy and thirsty and can't make it home. You check her blood pressure and find it to be LOW. What could you do right in your home to raise her blood pressure? A. Place a cold washcloth on her head B. Have her drink a large glass of cool water C. Have her take a shower with warm water D. Encourage her to take slow, deep breaths

B. Have her drink a large glass of cool water

Which example best describes metaplasia? A. In high altitudes, when there is an increase in the number of red blood cells to maximize the hemoglobin oxygen-carrying capacity. B. In GERD, when esophageal cells change from a squamous epithelium cell type to a glandular cell type. C. In the uterus, when there is an increase in hormone levels during menstrual cycle that cause an increased the number of cells in the uterus. D. In BPD, when stressors prompt cellular alterations that lead to chronic, irreversible cellular changes.

B. In GERD, when esophageal cells change from squamous epithelium cell type to a glandular cell type

You get a paper cut and experience pain at the site. This response is related to: A. Increased perfusion at the site B. Increased exudate and chemical mediators at the site C. Bacteria that have entered the wound D. Vasoconstriction at the site

B. Increased exudate and chemical mediators at the site

A client has an increase in her anion gap (AG). What does the nurse determine is the significance of this finding? A. It indicates the client has metabolic alkalosis. B. It indicates the client has metabolic acidosis. C. It indicates the client has respiratory acidosis. D. It indicates the client has respiratory alkalosis.

B. It indicates metabolic acidosis

The health care provider is performing a spinal tap on a client with suspected infection. The provider would perform the procedure at: A. T3 or T4 B. L3 or L4 C. C3 or C4 D. S3 or S4

B. L3 or L4

Treatment of an altered immune response with corticosteroids is associated with which one of the following adverse effects? A. Decreased blood sugar B. Loss of bone mineral C. Thickening of skin D. Weight loss

B. Loss of bone mineral

A client who has just recently been diagnosed with cancer asks the nurse what tumor markers are. Which answer would be the nurse's best response? A. Markers are doses of vitamins used to fight cancer. B. Markers are antigens expressed on the surface of tumor cells. C. Markers are marks the physician places to show the site of the cancer. D. Markers are enzymes released by the body to prevent cancer.

B. Markers are antigens expressed on the surface of tumor cells

The nurse is caring for a client who has excessive diarrhea. Which acid-base disturbance does the nurse anticipate will result from having excessive diarrhea? A. An increase in bicarbonate B. Metabolic acidosis C. Increased pH value D. No change in values from normal

B. Metabolic acidosis

For which acid-base imbalance will the nurse monitor for a client taking large doses of loop diuretics? A. Respiratory acidosis B. Metabolic alkalosis C. Respiratory alkalosis D. Metabolic acidosis

B. Metabolic alkalosis

The nurse is assessing a client for early manifestations of hyponatremia. The nurse would assess the client for: A. Peaked T wave on the EKG B. Muscle weakness C. Tachycardia D. Dry, sticky mucous membranes

B. Muscle weakness

A client had a positive Pap smear. The surgeon diagnosed "cancer in situ of the cervix." The client asks, "What does this mean?" From the following statements, which is most appropriate in response to this question? The tumor has: A. Been walled off within a strong fibrous capsule. B. Not crossed the basement membrane, so it can be surgically removed with little chance of growing back. C. Grown undifferentiated cells that no longer look like the tissue from which it arose. D. Developed a distant infiltration.

B. Not crossed the basement membrane, so it can be surgically removed with little chance of growing back

Which of the following transport mechanisms is associated with movement of water across a semipermeable membrane? A. Diffusion B. Osmosis C. Facilitated diffusion D. Active transport

B. Osmosis

Rheumatoid arthritis results in joint immobility as a result of: A. Synovial fluid loss B. Pannus formation C. Rheumatoid factor D. Joint deviation

B. Pannus formation

The study of functional alterations in human health because of an injury, disease, or syndrome describes which of the following? A. Pathology B. Pathophysiology C. Physiology D. Morphology

B. Pathophysiology

The transport mechanism requiring energy is: A. Facilitated diffusion B. Primary active transport C. Osmosis D. Diffusion

B. Primary active transport

The nurse is providing a prenatal class for a group of women at the local women's center. The nurse informs the group about the importance of taking their folic acid supplements for the prevention of neural tube defects. What type of prevention is the nurse providing? A. Initial prevention B. Primary prevention C. Tertiary prevention D. Secondary prevention

B. Primary prevention

A client complains of general malaise and fatigue and has a mild fever. The nurse would evaluate this stage of disease as the: A. Resolution stage B. Prodromal stage C. Incubation stage D. Convalescent stage

B. Prodromal stage

Disruption of the cell wall of a bacterium by antibacterial drugs will interrupt: A. Phagocytosis B. Protein and DNA synthesis C. Adherence D. Motility

B. Protein and DNA synthesis

What is the most likely complication of an untreated UTI? A. Glomerulonephritis B. Pyelonephritis C. Fulminant UTI D. Urethritis

B. Pyelonephritis

Total obstruction of the airway by aspirated material is manifested by: A. Hoarse cough B. Rapid loss of consciousness C. Dyspnea D. Inflammation of the mucosa

B. Rapid loss of consciousness

The nurse documents which assessment data as a symptom? A. Elevated temperature B. Report of pain C. Enlarged lymph node D. Pinpoint pupil size

B. Report of pain

The nurse enters a client's hospital room and finds the client breathing rapidly, stating, "I must be having a stroke, my fingers are tingling!" Which acid-base balance disorder is this client experiencing due to hyperventilation? A. Respiratory acidosis B. Respiratory alkalosis C. Metabolic alkalosis D. Metabolic acidosis

B. Respiratory alkalosis

The patient is diagnosed with influenza. Which of the following best describes the route of transmission for this disease? A. Airborne B. Respiratory droplets C. Direct contact D. Fecal-oral

B. Respiratory droplets

You decide that it has been too long since your last physical examination, so you schedule an appointment for a routine health screening. You have a blood cholesterol level checked and it is within the expected range. This activity represents which level of prevention? A. Primary prevention B. Secondary prevention C. Tertiary prevention D. None of these

B. Secondary prevention

Which condition is a complication of infection in which pathogens gain access to the blood? A. Chronic infection B. Septicemia C. Septic shock D. Bacteremia

B. Septicemia

Which of the following ions is most closely related to water movement? A. Potassium B. Sodium C. Chloride D. Calcium

B. Sodium

A client has been admitted to the intensive care unit with a myocardial infarction. After the client recovers from the acute course of the event and ready for discharge, the nurse provides information about the beta adrenergic blocker, atenolol, that the client will take to prevent complications after the MI. What type of prevention is the nurse providing? A. Initial prevention B. Tertiary prevention C. Secondary prevention D. Primary prevention

B. Tertiary prevention

You are listening to your lab partners heart in skills lab and you hear the "lub dub" sound. What are you hearing? A. The sodium ions rapidly influxing into the cells B. The closure of the heart valves C. The contraction of the left ventricle D. Blood forcing its way through the aorta

B. The closure of the heart valves

In an ICU setting, one assessment that would lead the nurse to suspect that shock has led to decreased blood flow to vital organs is: A. Blood pressure staying in the 98/72 range for the past hour. B. Urine output less than 20 mL/hour. C. Sleepiness and difficulty to arouse without using painful stimuli. D. Warm legs with peripheral vasodilation.

B. Urine output less than 20 mL/hour

The following season, you are concerned about getting the flu again. Which of the following statements is true? A. You continue to be at risk because nothing can protect you from reinfection B. Vaccination for prevalent strains of influenza virus can provide improved protection against the disease C. Premedication with immunosuppressants will provide protections against infection D. Because you have had the flu once, you will be protected from getting its again

B. Vaccination for prevalent strains of influenza virus can provide improved protection against the disease

A client tells the nurse that he is concerned he may be developing chronic bronchitis and asks how the diagnosis is made. The most appropriate information for the nurse to provide would be: A. A diagnosis of chronic bronchitis requires a history of a respiratory infection that occurs every 3 consecutive months in at least 2 consecutive years. B. A diagnosis of chronic bronchitis requires a chest x-ray that shows a tubercle bacillus that has been present for at least 2 consecutive years. C. A diagnosis of chronic bronchitis requires a history of a chronic productive cough that has persisted for at least 3 consecutive months in at least 2 consecutive years. D. A diagnosis of chronic bronchitis requires you to have been hospitalized at least one time in at least 2 consecutive years with shortness of breath.

C. A diagnosis of chronic bronchitis requires a history of a chronic productive cough that has persisted for at least 3 consecutive months in at least 2 consecutive years.

The health care provider is reviewing diurnal variation pattern in adrenocorticotropic (ACTH) levels. Select the typical diurnal variation pattern in adrenocorticotropic (ACTH) levels. A. ACTH increases in the morning and peaks again in the evening hours. B. ACTH maintains a consistent level regardless of the time of day. C. ACTH peaks in the morning and declines throughout the day. D. ACTH peaks in correspondence with food intake.

C. ACTH peaks in the morning and declines throughout the day

The most common cause of endocrine disorders is: A. Surgical removal of endocrine glands B. Infection C. Adenomas D. Immunodeficiency

C. Adenomas

Which is not correct about an opportunistic infection? A. Chemotherapy or radiation can cause life-threatening opportunistic diseases. B. An opportunistic infection primarily affects an individual with a weakened immunity as a result of illness. C. An opportunistic infection primarily affects a host with a normal immunity. D. Fungi can cause life-threatening opportunistic diseases.

C. An opportunistic infection primarily affects a host with a normal immunity

Select the statement that best describes autoimmune disease. A. Autoimmune diseases represent an increase in self-tolerance that results in damage to body tissues by the immune system. B. Autoimmune diseases result from an overuse of antibiotics that causes damage to body tissues by the immune system. C. Autoimmune diseases represent a disruption in self-tolerance that results in damage to body tissues by the immune system. D. Autoimmune diseases represent overuse of the immune system that causes damage to the body tissues.

C. Autoimmune diseases represent a disruption in self-tolerance that results in damage to body tissues by the immune system.

A client with a 25-year history of smoking is diagnosed with emphysema. Physical assessment reveals an increased anterior-posterior chest diameter. Which term should the nurse use to document this finding? A. Pink puffer B. Pneumothorax C. Barrel chest D. Blue bloater

C. Barrel chest

Which type of spinal cord injury is associated with the clinical manifestation of ipsilateral loss of motor and sensory function? A. Anterior cord syndrome B. Central cord syndrome C. Brown-Séquard syndrome D. Complete cord transection

C. Brown-Sequard syndrome

The nurse is caring for a client with a spinal cord injury. Assessment reveals shallow breath sounds with a very weak cough effort. The nurse correlates this with which level of injury on the spinal column? A. T10 B. T1 C. C5 D. C2

C. C5

The health care provider is reviewing diagnostic results of a client with suspected ovarian cancer. Select the result that would be of most concern. A. Human chorionic gonadotropin (hCG) B. Carcinoembryonic antigen (CEA) C. CA 125 D. Prostate-specific antigen (PSA)

C. CA 125

A client presents to the urgent care and tells the health care provider he came home from vacation 4 days ago and does not feel well. Upon assessment, the provider notes the client's right upper thigh area is red, warm, and painful, and the lymph nodes are palpable. The client states he had been swimming in a lake all week. The health care provider recognizes these manifestations as: A. Impetigo B. Scabies C. Cellulitis D. Rosacea

C. Cellulitis

Which one of the following is a diagnostic test for cervical dysplasia? A. Liquid-based Pap cytology B. Conventional Pap cytology C. Cervical punch biopsy D. HPV testing

C. Cervical punch biopsy

The changes seen in cells adapting to stressors that promote metaplasia: A. Are irreversible B. Can result in cancer C. Change from one cell type to another D. Show abnormal differentiation

C. Change from one cell type to another

Cellular atrophy results in A. Increase in tissue volume due to increase in cell number B. Increase in tissue volume due to increase ion cell size C. Decrease in tissue volume due to decrease in cell size D. No change in tissue volume

C. Decrease in tissue volume due to decrease in cell size

A nurse in the emergency department admits a male client who has experienced severe frostbite to his hands and toes after becoming lost on a ski hill. The nurse recognizes that which phenomena has contributed to his tissue damage? A. Decreased blood viscosity has resulted in interstitial bleeding. B. Autonomic nervous stimulation has resulted in injury. C. Decreased blood flow has induced hypoxia. D. Reactive vasodilation has compromised perfusion.

C. Decreased blood flow has induced hypoxia

Immune suppression in AIDS is related to: A. Decreased platelet count B. Decreased red blood cell count C. Decreased lymphocyte count D. Elevated lymphocyte count

C. Decreased lymphocyte count

Diabetes insipidus, if left untreated, will rapidly develop into: A. Malignant hypertension B. Diabetic coma C. Dehydration D. Metabolic alkalosis

C. Dehydration

A number of fungi, such as ringworm, athlete's foot, and jock itch, are incapable of growing at a core body temperature. Hence, their infection is limited to cooler cutaneous surfaces. What are these pathogens known as? A. Parasites B. Superficial mycoses C. Dermatophytes D. Systemic mycoses

C. Dermatoophytes

What is the process that makes cells with the same genetic material develop into specific cell types? A. Stimulation B. Proliferation C. Differentiation D. Reproduction

C. Differentiation

Carbon dioxide and oxygen movement within the cell occur based on which concept? A. Facilitated diffusion B. Osmosis C. Diffusion D. Active transport

C. Diffusion

Select the option that best identifies how adult cancers differ in origin from childhood cancers. Adult cancers originate from: A. The hematopoietic system. B. Embryonic cells. C. Epithelial cells. D. Neuronal tissues.

C. Epithelial cells.

Which of the following is the most common cause of acute pancreatitis? A. Cancer B. Autoimmunity C. Excess alcohol intake D. Cystic fibrosis

C. Excess alcohol intake

Seizure disorders are associated with which type of injury mechanism? A. Traumatic injury B. Pressure injury C. Excitation injury D. Ischemic injury

C. Excitation injury

Which of the following may make a person more susceptible to getting an infection? A. Age between 6 and 46 years B. Experiencing a surgery that is healing by primary intention C. Final exams week D. A functioning immune system

C. Final exams week

A client who is diagnosed with seizures describes feeling confused after experiencing a seizure. The family members report that the client has been smacking his lips prior to having a seizure. The client most likely experienced which type of seizure? A. Clonic B. Atonic C. Focal D. Myoclonic

C. Focal

A patient presents with exophthalmos. What condition is she likely experiencing? A. Addison disease B. Cushing disease C. Graves disease D. Diabetes insipidus

C. Graves disease

Abnormal stimulation of the thyroid gland by TSH-receptor antibodies is implicated in cases of: A. Addison disease B. Cushing disease C. Graves disease D. Cushing syndrome

C. Graves disease

The nurse is caring for a client with right upper quadrant pain secondary to acute choledocholithiasis. If the common bile duct becomes obstructed, which manifestation will the nurse expect? A. Vomiting B. Hemorrhage C. Hyperbilirubinemia D. Ascites

C. Hyperbilirubinemia

Hypotonic fluid loss may result in A. Hyponatremic dehydration B. Isonatremic dehydration C. Hypernatremic dehydration D. None of the above

C. Hypernatrmic dehydration

The nurse is caring for a client who is 1 day postoperative for a thyroidectomy. The client complains of tingling of the hands and feet and around his mouth. The nurse suspects these symptoms are a manifestation of which electrolyte disturbance? A. Hyponatremia B. Hypercalcemia C. Hypocalcemia D. Hypophosphatemia

C. Hypocalcemia

The nurse is providing care for a client whose abrupt-onset hypersensitivity reaction was stimulated by an antigen challenge. The nurse should understand that this reaction is: A. Complement-mediated. B. Antibody-mediated. C. IgE-mediated. D. T-cell-mediated.

C. IgE mediated

A nurse is reviewing labs for a client newly diagnosed with a bacterial infection. In determining if the client is experiencing a primary immune response, the nurse looks for which type of antibody produced from activated immature B cells? A. IgA B. IgE C. IgM D. IgG

C. IgM

The health care team is developing a plan of care for a client diagnosed with congestive heart failure (CHF). The primary treatment goal would be: A. Maintaining higher oxygen levels to decrease the work of breathing. B. Eliminating CHF through curing the disease. C. Improving quality of life by relieving symptoms. D. Placing a stent for fluid drainage from the heart.

C. Improving quality of life by relieving symptoms

Which of the following is the most common cause of acute gastritis? A. Poor gastric perfusion B. Too much stomach acid C. Ingestion of aspirin, alcohol, or other chemicals D. H. pylori infection

C. Ingestion of aspirin, alcohol, or other chemicals

You are caring for a female patient who has reported a noticeable decrease in breast size and muscle mass. Which of the following conditions and causes is the most likely explanation? A. Puberty B. Pregnancy C. Menopause D. Acromegaly

C. Menopause

A client presents to the emergency department with some vague symptoms. After history and physical exam, the physician is suspecting the client may have viral hepatitis. Which clinical manifestation leads the nurse to suspect the client is in the prodromal period of viral hepatitis? A. Slight jaundice in the sclera of the eyes B. Liver tenderness on palpation C. Muscle aches and pain along with fatigue D. Onset of severe itching with skin breakdown

C. Muscle aches and pain along with fatigue

Which change exemplifies physiologic hypertrophy? A. Cell size increase with hypoxia B. Lung size increase in emphysema C. Muscle mass increase with exercise D. Heart size increase in hypertension

C. Muscle mass increases with exercise

During a lecture on inflammation, the physiology instructor discusses the major cellular components involved in the inflammation response. The instructor asks, "Which cells arrive early in great numbers?" Which student response is correct? A. Basophils B. Monocytes C. Neutrophils D. Lymphocytes

C. Neutrophils

Emphysema differs from chronic bronchitis in that emphysema: A. Is characterized by mucous production and inflammation B. Obstructs the large airways C. Obstructs the alveoli D. There are no differences between the two conditions

C. Obstructs the alveoli

Cell death by necrosis is: A. The cell's way of replacing aged cells with new cells B. Also known as programmed cell death C. Often a response to inflammation D. Commonly seen during the period of embryo development

C. Often a response to inflammation

Which type of cancer would most likely spread through the process called seeding? A. Lung B. Colon C. Ovarian D. Bone

C. Ovarian

An older adult client asks the nurse what causes the functional decline that occurs with the process of aging. The best response would include which physiologic mechanism? A. Hypercalcemia B. Endothelial dysfunction C. Oxidative stress D. Hypoxia

C. Oxidative stress

Which of the following is least likely to be a part of the daily routine for a patient with severe asthma? A. Inhaled bronchodilators B. Inhaled corticosteroids C. Oxygen therapy D. Avoiding triggers

C. Oxygen therapy

Cancer is on the rise across the globe. Which term describes this phenomenon? A. Epidemic B. Endemic C. Pandemic D. Morbidity

C. Pandemic

The nurse is caring for a client exhibiting bronchoconstriction. Which division of the nervous system contributes to this finding? A. Afferent B. Somatic C. Parasympathetic D. Sympathetic

C. Parasympathetic

A woman experiences a viral infection while pregnant. Which type of immunity does an infant have at birth against this infection? A. Tolerance B. Active C. Passive D. Adaptive

C. Passive

The transfer of secretory IgA from mother to infant during breastfeeding is an example of A. Hypersensitivity reaction B. Active immunity C. Passive immunity D. Alloimmunity

C. Passive immunity

The nurse notifies the health care prescriber of a client's serum potassium level of 6.2 mEq/L (6.2 mmol/L). Which prescription will the nurse consider as having the first priority? A. Administering IV potassium chloride B. Administering sodium polystyrene sulfonate C. Placing the client on a cardiac monitor D. Placing the client on a potassium-restricted diet

C. Placing the client on a cardiac monitor

The feeling that "something is not quite right" is considered which stage in infection? A. Point of infection with pathogen B. Incubation C. Prodrome D. Acute symptoms

C. Prodrome

A client with multiple sclerosis has been asymptomatic for three months. How is the client's condition described? A. Recurrent B. Relapsing C. Remission D. Resolved

C. Remission

Your patient is experiencing peripheral edema, hepatomegaly, ascites, and splenomegaly. Which of the following conditions would be consistent with the patient's findings? A. Endocarditis B. Myocardial infarction C. Right-sided heart failure D. Left-sided heart failure

C. Right-sided heart failure

A client has a burn that involves the entire epidermis and various degrees of the dermis. It is painful, moist, and blistered. The nurse recognizes the burn as: A. Third-degree full thickness B. Second-degree full thickness C. Second-degree partial thickness D. First-degree partial thickness

C. Second degree partial thickness

A client who has been taking 80 mg of prednisone, a glucocorticoid, each day has been warned by his primary care provider to carefully follow a plan for the gradual reduction of the dose rather than stopping the drug suddenly. What is the rationale for this directive? A. Stopping the drug suddenly may "shock" the HPA axis into overactivity. B. Sudden changes in glucocorticoid dosing may reverse the therapeutic effects of the drug. C. Stopping the drug suddenly may cause acute adrenal insufficiency. D. Sudden cessation of a glucocorticoid can result in adrenal gland necrosis

C. Stopping the drug suddenly may cause acute adrenal insufficiency

The nurse is explaining to a client's family how vasogenic brain edema occurs. The mostappropriate information for the nurse to provide would be: A. There is an increase in the production of cerebrospinal fluid volume. B. Normal physiologic circumstances result in decreased adsorption of CSF. C. The blood-brain barrier is disrupted, allowing fluid to escape into the extracellular fluid. D. There is a decrease in the amount of fluid volume in the brain.

C. The blood-brain barrier is disrupted, allowing fluid to escape into the extracellular fluid

A pediatric nurse practitioner is performing a comprehensive assessment of a 9-year-old girl. What assessment finding most warrants further assessment and follow-up? A. The girl has not been to the dentist in 5 months. B. The girl has grown 4 cm over the past 12 months. C. The girl has gained 7 kg over the past 6 months. D. The girl has not received her annual influenza vaccination.

C. The girl has gained 7 kg over the past 6 moths

A reduced number of erythrocytes (RBCs) in the blood results in the following change in the oxygen saturation (SaO2) of the blood: A. The SaO2 would increase B. The SaO2 would decrease C. The number of RBCs will not affect the SaO2 D. There will be a decrease only if the osmotic pressure of the blood is also decreased

C. The number of RBCs will not affect the SaO2

A client has died and the nurse is responsible for filling out the death certificate. In performing this task, the nurse is required to record the client's age, sex, and cause of death, among other factors. What is the purpose of reporting these statistics? A. These statistics are recorded so pharmaceutical companies may compile the data to determine future need. B. The government requires the statistics in order to calculate the crude mortality rate. C. These statistics are useful in terms of anticipating health care needs, planning public-education programs, directing health research efforts, and allocating health care dollars. D. These statistics are required by the hospital to inform the WHO for the coding of the cause of death.

C. These statistics are useful in terms of anticipating health care needs, planning public-education programs, directing health research efforts, and allocating health care dollars.

A newly introduced diagnostic test has been found to produce unacceptably low levels of reliability. The nurse should recognize that the test: A. Is impractical to implement in daily practice. B. Does not actually measure what it purports to measure. C. Yields inconsistent results when repeated. D. Does not reflect the status of the population as a whole.

C. Yields inconsistent results when repeated

A baseball player was hit in the head with a bat during practice. In the emergency department, the physician tells the family that he has a "coup" injury. How will the nurse explain this to the family so they can understand? A. "When the bat hit his head, his neck jerked backward causing injury to the spine." B. "Your son has a huge laceration inside his brain where the bat hit his skull." C. "Your son has a contusion of the brain at the site where the bat hit his head." D. "It's like squeezing an orange so tight that the juice runs out of the top."

C. Your son has a contusion of the brain at the site where the bat hit his head

A 10 year old child with strep throat asks the nurse, "Why are there large bumps [lymph nodes] on my neck when my throat gets sore?" The nurse replies that lymph nodes: A. Help your tonsils get bigger with cells that will bring immune cells into your throat to prevent any other infections. B. Bring in cells into the lymph node [your bump] to stop the germs from going anywhere else in the body. A. Help your body fight off infections by allowing special cells [lymphocytes and macrophages] to move through the lymph chain and engulf and destroy germs. D. Bring all kind of good cells to your throat so that they can wall the strep off and keep the germs from getting any food or water.

C. help your body fight off infections by allowing special cells [lymphocytes and macrophages] to move through the lymph chain and engulf and destroy germs.

When a 55-year-old client's routine blood work returns, the nurse notes that the client's C-reactive protein (CRP) is elevated. The client asks what that means. The nurse responds: A. "This means you have high levels of HDL to balance the LDL found in animal proteins." B. "You are consuming high levels of folate, which works with the B vitamins and riboflavin to metabolize animal protein." C. "You must eat a lot of red meat since this means you have a lot of fat floating in your vessels." D. "This means you have elevated serum markers for systemic inflammation that has been associated with vascular disease."

D. "This means you have elevated serum markers for systemic inflammation that has been associated with vascular disease."

Which individual is experiencing a health problem that is the result of a parasite? A. A college student who contracted Chlamydia trachomatis during an unprotected sexual encounter B. A woman who developed hepatitis A from eating at an unhygienic restaurant C. A hospital client who has developed postoperative pneumonia D. A man who acquired malaria while on a tropical vacation

D. A man who acquired malaria on a tropical vacation

Neurons are characterized by the ability to communicate with other neurons and body cells through pulsed electrical impulses, or: A. Astrocytes. B. Axon hillocks. C. Nodes of Ranvier. D. Action potentials.

D. Action potentials

Metabolic acidosis may be associated with A. Increased chloride levels B. Increased metabolic acids C. Decreased bicarbonate D. All of the above

D. All of the above

Which is a major cause of respiratory failure? A. Aspiration B. Atelectasis C. Sepsis D. All of these

D. All of these

Which of the following can trigger acute respiratory distress syndrome? A. Severe lung infection B. Inhaling toxic fumes C. Aspirating stomach contents into the lungs D. All of these

D. All of these

Which of the following contributes to evidence-based practice and high-quality patient care? A. Primary research B. Clinical expertise C. Meta-analysis studies D. All of these contribute

D. All of these contribute

Which condition is an example of wound healing by secondary intention? A. Sacral skin tear closed with Steri-Strip B. Abdominal wound with staples C. Leg laceration with sutures D. An infected burn of the arm

D. An infected burn on the arm

The nurse is caring for a client with chronic pancreatitis who reports abdominal pain. Which location will the client's pain radiate? A. Groin B. Chest C. Shoulder D. Back

D. Back

A teenager has been in a car accident and experienced an acceleration-deceleration head injury. Initially, the client was stable but then started to develop neurological signs/symptoms. The nurse caring for this client should be assessing for which type of possible complication? A. Status epilepticus B. TIAs and cerebrovascular infarction C. Momentary unconsciousness D. Brain contusions and hematomas

D. Brain contusions and hematomas

A client with gastroesophageal reflux disease has metaplasia. Which explanation is the cause? A. These are nonreversible cell changes. B. This represents cancerous cells. C. Cells are increased in size due to increased oxygenation. D. Cells are replaced in response to chronic irritation.

D. Cells are replaced in response to chronic irritation

Which intracranial volume is most capable of compensating for increasing intracranial pressure? A. Brain cell tissue B. Intravascular blood C. Surface sulci fluid D. Cerebrospinal fluid

D. Cerebrospinal fluid

When considering the clinical course, a disease that is characterized by remissions and exacerbations is considered to be: A. Preclinical. B. Acute. C. Subclinical. D. Chronic.

D. Chronic

A client reports fatigue and an inability to concentrate. The client has been unemployed for the second time in a year and is concerned about supporting the family. Select the type of stressor the client most likely is experiencing. A. Associated B. Acute time-limited C. Acute reflexive D. Chronic sustained

D. Chronic sustained

Autoimmunity may be triggered by which one of the following? A. Elimination of self-reactive lymphocytes in central lymphoid tissues B. Persistent lymphocyte ignorance C. Impaired T-cell activation D. Close resemblance between foreign and self-antigen

D. Close resemblance between foreign and self-antigen

A client is immobilized following a hip injury. The client has developed lower leg discoloration with edema, pain, and tenderness in the midcalf area. How should the nurse document these clinical findings? A. Arterial insufficiency B. Primary varicose veins C. Stasis ulcerations D. Deep vein thrombosis

D. Deep vein thrombosis

A woman reports to the nurse that she has developed a yeast infection. The woman does not understand how she could get a yeast infection since she has been on antibiotics for a urinary tract infection. What is the rationale for this client's complaint? A. Antibiotics allow yeast to access sterile environments in the body. B. Yeast prefers a warm, moist, and dark environment, such as that present in the female perineum. C. Yeast grows well when exposed to sugar, which is found as a carrier substance in most antibiotics. D. Destroying one type of resident flora (bacteria) can allow overproliferation of another competing type (yeast).

D. Destroying one type of resident flora can allow over proliferation of another competing yeast

Which of the following is true regarding pharmacologic treatment for AIDS? A. Drugs are used to target increased red blood cell number B. Drugs are used to target increased white blood cell number C. Drugs are used to increase host DNA replication D. Drugs are used to inhibit HIV replication

D. Drugs are used to inhibit HIV replication

A nurse on a medical unit is providing care for a 37-year-old female client who has a diagnosis of Graves disease. Which assessments should the nurse prioritize? A. Cognition and judgment B. Skin integrity and distribution of adipose tissue C. Signs and symptoms of decreased bone density D. Eye health and visual acuity

D. Eye health and visual acuity

In evaluating modifiable cardiovascular risk factors for your patient, which one is NOT considered modifiable? A. Poorly controlled diabetes mellitus B. Hyperlipidemia C. Hypertension D. Female gender

D. Female gender

A client with malignant hypertension is at risk for a hypertensive crisis, including the cerebral vascular system often causing cerebral edema. The nurse would assess this client for which signs and symptoms? A. Stupor and hyperreflexia B. Papilledema and lethargy C. Restlessness and nervousness D. Headache and confusion

D. Headache and confusion

A client fell off his motorcycle, receiving several large abrasion-related surface wounds. What physiologic phenomenon will the client first experience? A. Healing by primary intention B. Maturation C. Remodeling D. Healing by secondary intention

D. Healing by secondary intention

A client has recently received a pneumococcal vaccine and the client's B cells are consequently producing antibodies. Which cells may enhance this production of antibodies? A. Cytotoxic T cells B. Natural killer cells C. Regulatory T cells D. Helper T cells

D. Helper T cells

The nurse is teaching a client diagnosed with Addison disease about the importance of lifetime oral replacement therapy. Which pharmacologic agent would be the drug of choice and included in this teaching plan? A. Insulin B. Potassium supplements C. Ketoconazole D. Hydrocortisone

D. Hydrocortisone

Which of the following reactive oxygen species is scavenged by catalase? A. Hydroxyl radical B. Peroxynitrite C. Superoxide D. Hydrogen peroxide

D. Hydrogen peroxide

A nurse observes peaked, narrow T waves on the electrocardiogram of a client suffering from renal failure. The nurse suspects that the client is experiencing which condition? A. Hypernatremia B. Hypokalemia C. Hyponatremia D. Hyperkalemia

D. Hyperkalemia

The nurse explains to the hypertensive client that the increased workload required to pump blood against an elevated arterial pressure results in a progressive increase in left ventricular muscle mass. This is an example of: A. Hyperplasia. B. Dysplasia. C. Metaplasia. D. Hypertrophy.

D. Hypertrophy

You are experiencing constipation, dry skin, weight gain, and cold intolerance. Which condition are you most likely experiencing? A. Hyperthyroidism B. Addison disease C. Cushing syndrome D. Hypothyroidism

D. Hypothyroidism

Three days ago, a mother delivered her full-term infant who had been identified as having an in utero infection. The infant is receiving antibiotic and phototherapy, and the mother is breast-feeding. Which types of immunoglobulins could most reasonably be expected to predominate in the infant's immune system? A. IgM, IgD, IgM B. IgA, IgM, IgD C. IgE, IgG, IgD D. IgG, IgA, IgM

D. IgG, IgA, IgM

The pathology related to systemic lupus erythematosus is due to: A. Neutrophil activation B. Delayed immunity C. Immunosuppression D. Immune complex deposition

D. Immune complex deposition

Which physiologic process best exemplifies a positive feedback mechanism? A. Release of parathyroid hormone in response to decreased serum calcium levels B. Release of antidiuretic hormone when sodium levels are higher than normal C. Regulation of blood glucose levels by insulin D. increase in prolactin secretion that occurs with more frequent breastfeeding

D. Increase in prolactin secretion that occurs with more frequent breastfeeding

While lecturing to a group of physiology students, the instructor asks, "What metabolic factors cause vasodilation of cerebral vessels, thereby increasing cerebral blood flow to the brain?" The best student response would be: A. Decreased serum sodium level B. Increased oxygen saturation C. Decreased hydrogen ion concentration D. Increased carbon dioxide level

D. Increased carbon dioxide level

A client with a history of emphysema is experiencing hypoxemia after a taxing physical therapy appointment. Which physiologic phenomenon will occur as a consequence of hypoxemia? A. Necrosis B. Hypoventilation C. Peripheral vasodilation D. Increased heart rate

D. Increased heart rate

A client comes to the clinic with fatigue and muscle weakness. The client also states she has been having diarrhea. The nurse observes the skin of the client has a bronze tone and when asked, the client says she has not had any sun exposure. The mucous membranes of the gums are bluish-black. When reviewing laboratory results from this client, what does the nurse anticipate seeing? A. Positive C-reactive protein B. Increase in sedimentation rate C. Elevated WBC count D. Increased levels of ACTH

D. Increased level of ACTH

The nucleus is called the center of the cell because it has the ability to do which of the following? A. It transforms organic compounds into energy that is accessible to the cell. B. It breaks down and digests worn-out parts of the cell. C. It carries and copies DNA instructions for protein synthesis to the cytoplasm. D. It contains the DNA that is essential for protein synthesis to keep the cell alive.

D. It contains the DNA that is essential for protein synthesis to keep the cell alive

Which of the following is an example of a strong acid? A. Albumin B. Inorganic phosphorus C. Sodium D. Lactate

D. Lactate

Which is not a local manifestation of acute inflammation? A. Edema B. Redness C. Loss of function D. Leukocytosis

D. Leukocytosis

Feelings of dread, high anxiety, or exquisite pleasure can be elicited by stimulation of areas in which structure? A. Temporal lobe B. Cerebellum C. Occipital lobe D. Limbic system

D. Limbic system

The living part of the cell found both inside and outside of the nucleus is called protoplasm. In addition to proteins, carbohydrates, electrolytes, and water, what other substance is found in protoplasm? A. Hydrolytic enzymes B. Peroxisomes C. Acid hydrolases D. Lipids

D. Lipids

Which example best represents pathogenesis of disease? A. Venous thrombosis leading to myocardial infarction B. The rate of influenza spread at any given time C. Increased rates of stroke in those with hypertension D. Liver cell destruction occurring long before cirrhosis of the liver

D. Liver cell destruction occurring long before cirrhosis of the liver

A nursing student who is studying cancer cells identifies which of the following to be the bestdefinition of anaplasia? A. Decreased proliferation of cancer cells B. Rapid proliferation of cancer cells C. Increased cell differentiation in cancerous tissue D. Loss of cell differentiation in cancerous tissue

D. Loss of cell differentiation in cancerous tissue

The organelle that is involved in cellular respiration and is linked to the development of oxidative stress is known as the: A. Endoplasmic reticulum B. Golgi apparatus C. Lysosome D. Mitochondria

D. Mitochondria

You are looking to break the chain of infection by washing your hands frequently as you provide care for patients. Which of the following links in the chain will be broken by this activity? A. Reservoir B. Host C. Portal of entry D. Mode of transmission

D. Mode of transmission

The nurse is caring for a client with a longstanding diagnosis of hypocalcemia secondary to kidney disease. The nurse observes which clinical manifestations in this client? A. Loss of appetite and reports of nausea B. Lethargy and change in level of consciousness C. High fluid intake and copious amounts of dilute urine output D. Muscular spasms and reports of tingling in hands/feet

D. Muscular spasm and reports of tingling in hands/feet

Which peripheral nerve injury will likely result in cellular death with little chance of regeneration? A. Cutting injury where slow-regeneration axonal branches are located B. Incomplete amputation where tubular implants are used to fill in the gaps of nerves C. Crushing injury where the nerve is traumatized but not severed D. Nerve fibers destroyed close to the neuronal cell body

D. Nerve fibers destroyed close to the neuronal cell body

The lobe of the brain primarily involved in functions related to vision is the: A. Frontal lobe B. Parietal lobe C. Temporal lobe D. Occipital lobe

D. Occipital lobe

What is the one definitive test to diagnose rheumatoid arthritis? A. A positive rheumatoid factor (RF) B. An elevated erythrocyte sedimentation rate (ESR) C. A positive antinuclear antibody (ANA) D. One test is not definitive

D. One test is not definitive

A client with a diagnosis of community-acquired pneumonia is producing copious secretions that are physically obstructing the airway. Which pathophysiologic process will result from this condition? A. Dead air space B. Compensatory vasoconstriction C. Ventilation without perfusion D. Perfusion without ventilation

D. Perfusion without ventilation

Inflammation is ultimately needed to: A. Increase the inflammatory mediators at the site to vasoconstrict the area B. Increase platelets at the site for clotting C. Restore functional cells D. Prepare the site for healing

D. Prepare site for healing

A client is suspected of having the onset of alcoholic liver disease. The nurse should be assessing for which manifestation related to the necrosis of liver cells? A. Development of multiple skin nodules B. Long muscle group atrophy C. Tremors of the hands D. Rapid onset of jaundice

D. Rapid onset of jaundice

The renal control mechanism of restoring the acid-base balance is accomplished through which process? A. Reabsorption of hydrogen and excretion of carbonic acid B. Stimulation of the chemoreceptors in the brain stem C. Regulation of the production of carbonic acid D. Reabsorption of HCO3 and excretion of H+ restores acid-base balance through the renal control mechanisms.

D. Reabsorption of HCO3 and excretion of H+ restores acid-base balance through the renal control mechanisms.

A client arrives in the emergency department by ambulance with a family member stating, "He took an overdose of sleeping pills and I found him breathing very shallowly." For which type of acid-base disturbance will the nurse anticipate this client will be treated? A. Metabolic alkalosis B. Respiratory alkalosis C. Metabolic acidosis D. Respiratory acidosis

D. Respiratory acidosis

The nurse is caring for a client who complains of headache and blurred vision. The nurse recognizes that these symptoms, accompanied by increased plasma partial pressure carbon dioxide (PCO2) level and decreased pH level, are consistent with which diagnosis? A. Metabolic acidosis B. Metabolic alkalosis C. Respiratory alkalosis D. Respiratory acidosis

D. Respiratory acidosis

A patient presents with SIADH and a serum sodium level of 100 mEq/L. What would you expect for clinical manifestations for this patient? A. Polyuria, polydipsia, polyphagia B. Nausea, vomiting, headache C. Muscle cramps, weakness, irritability D. Seizures, psychosis, gait disturbances

D. Seizures, psychosis, gait disturbances

What statement is true concerning the use of ATP [adenosine triphosphate] by skeletal muscles? A. Skeletal muscles increase the use of ATP at rest. B. Skeletal muscles store ATP during exercise. C. Skeletal muscles do not use ATP for energy. D. Skeletal muscles use ATP at a rapid rate during exercise.

D. Skeletal muscles use ATP at a rapid rate during exercise

Inflammatory bowel disease (IBD) is used to designate two related inflammatory intestinal disorders: Crohn disease and ulcerative colitis. The nurse recognizes the difference between the distribution pattern between Crohn disease and ulcerative colitis. Which pattern describes Crohn's disease? A. Primarily rectum and colon involvement B. Continuous involvement of the colon starting at the rectum C. Development of cancer D. Skip lesions

D. Skip lesions

The nurse is caring for a client who is experiencing an increased level of aldosterone secretion. The nurse anticipates that the client may develop: A. Water and potassium retention. B. Potassium retention and water excretion. C. Potassium and sodium excretion. D. Sodium and water retention.

D. Sodium and water retention

A multidisciplinary healthcare team operates a program aimed at the prevention, identification, and treatment of diabetes in a rural community. Which aspect of the program would be most likely to be classified as secondary prevention? A. Teaching schoolchildren how a nutritious, traditional diet can lessen their chances of developing adult-onset diabetes. B. Administering oral antihyperglycemic medications to clients who have a diagnosis of diabetes. C. Regularly scheduled wound-dressing changes for clients who have foot ulcers secondary to peripheral neuropathy and impaired wound healing. D. Staffing a booth where community residents who are attending a baseball tournament can have their blood glucose levels checked.

D. Staffing a booth where community residents who are attending a baseball tournament can have their blood glucose levels checked.

A 42-year-old female client with breast cancer has a tumor that is minimal in size and extension, has minimal regional lymph nodes involved, and has no distant metastasis. Using the TNM system, the tumor is staged as: A. T1, N2, and M0 B. T1, N1, and M1 C. T0, N3, and M1 D. T1, N1, and M0

D. T1, N1, M0

A client newly diagnosed with cancer receives a poor prognosis. What is true regarding the client's clinical condition? A. The client has 6-8 months to live. B. The client has at least 12 months to live. C. The client has a decreased risk for morbidity. D. The client has an increased risk for mortality.

D. The client has an increased risk for mortality

A 77-year-old male client with a diagnosis of stomach cancer has been found to have metastases in his liver. The client and his family are surprised at this turn of events, stating that they don't see how he could have developed cancer in his liver. Which fact would underlie the reply that the care team provides? A. The proximity of the liver to the stomach allows for direct spread of cancerous cells due to a lack of contact inhibition. B. Hepatic stromal tissue shares characteristics with cancerous cells, including lack of anchorage dependence. C. The parenchymal tissue of the liver is particularly susceptible to secondary malignancies. D. The portal circulatory system brings venous blood from the GI tract into the liver.

D. The portal circulatory system brings venous blood from the GI tract into the liver

A college student has just received a positive HIV test result. How can the nurse most accurately interpret and respond to this finding? A. The student has HIV and may or may not develop AIDS. B. The student has HIV and will eventually develop AIDS. C. The student has AIDS and psychosocial help should be offered. D. The student has HIV antigens and further testing should be done.

D. The student has HIV antigens and further testing should be done

Which of the following does not affect diffusing capacity? A. The partial pressure of oxygen and carbon dioxide B. The alveolar surface area C. The density of the alveolar membrane D. The volume of air in the atmosphere

D. The volume of air in the atmosphere

Why are some diseases termed syndromes? A. They are a group of disease states that has the same etiology. B. They leave sequelae (e.g., lesions) and residual effects. C. They have complications. D. They are a compilation of signs and symptoms characteristic of a specific disease state.

D. They are a compilation of signs and symptoms characteristic of a specific disease state.

Which most important complication of atherosclerosis can result in occlusion of small heart vessels? A. Fibrous plaque B. Ulceration C. Fatty streaks D. Thrombosis

D. Thrombosis

Given the mode of transmission for influenza, how would you break the chain of infection and prevent spread? A. Administer antibiotics as directed B. Wash hands after toileting C. Disinfect table surfaces in the room D. Wear a mask

D. Wear a mask

An important aspect of infection is prevention. Which of the following would be an effective prevention measure for malaria? A. Antibiotics B. Avoiding international travel C. Handwashing D. Wearing long sleeves and pants

D. Wearing long sleeves and pants

The nurse is caring for a client who has just learned that he is HIV positive. The client asks the nurse how long he has been able to infect others. Select the best response by the nurse. A. "You can infect others only when you are actually diagnosed with AIDS." B. "You can infect others only after you have experienced seroconversion." C. "You can infect others when you start to get symptoms." D. "You can infect others before the HIV antibody is detectable in your blood."

D. You can infect others before the HIV antibody is detectable in your blood

The nurse is caring for a client with metabolic alkalosis. Which of these arterial blood gas results supports this diagnosis? A. pH of 7.45 and HCO3 of 24 mEq/L (24 mmol/L) B. pH of 7.25 and HCO3 of 18 mEq/L (18 mmol/L) C. pH of 7.35 and HCO3 of 22 mEq/L (22 mmol/L) D. pH of 7.50 and HCO3 of 45 mEq/L (45 mmol/L)

D. pH of 7.50 and HCO3 of 45 mEq/L

The nurse caring for a client with respiratory acidosis examines arterial blood gas (ABG) results. Which change from the initial value indicates the cause of the client's respiratory acidosis is improving? A. pH has decreased B. paO2 has decreased C. HCO3- has decreased D. paCO2 has decreased

D. paCO2 has decreased

Clients with chronic obstructive lung disease (COPD) may experience airway closure at the end of normal lung volumes instead of low lung volumes. This results in: A. Release of epinephrine, a catecholamine, which causes airway dilation. B. Airway constriction of the main bronchus. an increase in the physiologic dead space in alveoli that are C. Perfused but not ventilated. D. Trapping of large amounts of air that cannot participate in gas exchange.

D. trapping of large amounts of air that cannot participate in gas exchange.

A client has presented to the emergency department after he twisted his ankle while playing soccer. Which assessment findings are cardinal signs that the client is experiencing inflammation? Select all that apply. A. The client is experiencing pain B. The ankle is warmer than the unaffected ankle C. The client's ankle is visibly red D. The ankle appears to be swollen E. The ankle is bleeding

A,B,C,D

What type of viral hepatitis is not associated with transmission through contact with infected blood? A. A B. C C. B D. D

A. A

Which is not a requirement for effective perfusion? A. Absence of chronic disease B. Adequate blood volume C. Functional systemic circulation D. An open airway

A. Absence of chronic disease

A 79-year-old woman reports a recent onset of "nearly constant heartburn." During the assessment interview, she states that she has "lots of aches and pains." She states that she is not on any prescription medications but often takes aspirin for pain. The nurse should suspect what diagnosis? A. Acute gastritis B. Gastric cancer C. Helicobacter pylori infection D. Staphylococcal infection

A. Acute gastritis

A pH of 7.5 is defined as A. Alkalosis B. Acidosis C. Acidemia D. Alkalemia

A. Alkalosis

A client is applying to nursing school and has come to the clinic with a request to be tested for immunity to hepatitis B. Which type of testing would be best to determine immunity? A. Antibody titer B. DNA testing C. WBC count D. Culture

A. Antibody titer

A client has developed a tumor of the posterior pituitary gland. The client is at risk for problems with secretions of: A. Antidiuretic hormone (ADH) and oxytocin B. Somatostatin and prolactin C. Growth hormone-releasing hormone (GHRH) and dopamine D. Adrenocorticotropic hormone (ACTH) and vasopressin

A. Antidiuretic hormone and oxytocin

A client with a spinal cord injury at T8 would likely retain normal motor and somatosensory function of her: A. Arms B. Bladder C. Bowels D. Perineal musculature

A. Arms

During the admission interview the client, who is admitted with bacterial pneumonia, reveals a 20 pack per year smoking history. The nurse relates the possible cause of this pneumonia to the decreased defense of the pulmonary system caused by cigarette smoking. Smoking affects the pulmonary defense system in what way? A. Damage or destruction of cilia B. Vasodilation of blood vessels C. Increased development of atherosclerosis D. Buildup of nicotine on the cilia

A. Damage or destruction of cilia

The hospitalized burn patient wants to know why you need to remove his dressings every day. It is painful and he wants to avoid uncovering his burn injury. You explain that removing the dressings promotes: A. Debridement B. Infection C. Skin function D. Drying the exudate

A. Debridement

Which of the following are considered antigen-presenting cells? A. Dendritic cells B. Eosinophils C. B lymphocytes D. T lymphocytes

A. Dendritic cells

Which of the following meals would you recommend to a patient with a wound to promote healing? A. Eggs and orange juice B. Spaghetti and garlic toast C. Steak and potatoes D. Tomato soup and grilled cheese

A. Eggs and orange juice

Which of the following diseases is most likely the cause of your patient's barrel chest? A. Emphysema B. Pneumonia C. Tuberculosis D. Acute respiratory distress syndrome

A. Emphysema

Which of the following diagnostic tests would give information to the presence of a carrier-state asymptomatic hepatitis B infection? A. Hepatitis B surface antigen (HBsAg) B. Hepatitis B core antigen (HBcAg) C. Prothrombin time (PTT) D. There is no way to distinguish the different forms of hepatitis B

A. Hepatits B surface antigen (HBsAg)

A major manifestation of Cushing syndrome includes which of the following? A. Hypokalemia B. Excessive salt loss C. Hair and weight loss D. Muscle hypertrophy

A. Hypokalemia

Diagnosis of SIADH is based on which of the following? A. Hyponatremia B. Hypertonicity C. Increased urine volume D. Dilute urine with a low sodium content

A. Hyponatremia

An older adult female client who reports increasing fatigue has been diagnosed with aortic stenosis, a disease that her primary care provider believes may have been long-standing. Which compensatory mechanism has most likely maintained the woman's ejection fraction until recently? A. Left ventricular hypertrophy B. Increased blood pressure C. Aortic dilation D. Increased heart rate and stroke volume

A. Left ventricular hypertrophy

The nurse is caring for a client who is immunosuppressed. Which aspect of the client's pulmonary defense mechanism is affected? A. Macrophages in the alveoli B. Triggering a sneeze or cough reflex C. Warming and humidifying the air D. Signaling the respiratory control center

A. Macrophages in the alveoli

Hypokalemia is often associated with which one of the following conditions? A. Metabolic alkalosis B. Metabolic acidosis C. Hyperchloremia D. None of the above

A. Metabolic alkalosis

Treatment for tinea unguium must include: A. Oral antifungals B. Topical antifungals C. Both oral and topical antifungals D. Nail removal and topical antifungals

A. Oral antifungals

Before antibodies are detected in the humoral immune response, B cells differentiate into which type of cells during adaptive immunity process? A. Plasma B. Stem C. Cytotoxic D. Helper

A. Plasma

What is a common opportunistic infection in clients with AIDS? A. Pneumocystis jiroveci pneumonia (PCP) B. Chancroid C. Syphilis D. Myalgia

A. Pneumocystis jiroveci pneumonia (PCP)

Which assessment data would a nurse identify as a complication of a disease or disorder? A. Pulmonary emboli following deep vein thrombosis B. Hyperlipidemia in cardiovascular disease C. Weight gain of 1 to 2 pounds per week during pregnancy D. Reduced range of motion following a fracture

A. Pulmonary emboli following deep vein thrombosis

Which of the following clinical manifestations is not typically found with inflammation but is more characteristic of a bacterial infection? A. Purulent exudate B. Redness and swelling at the site C. Lymphadenopathy D. Fever

A. Purulent exudate

Hypokalemia is a hallmark of the following conditions? A. Salt-losing tubulopathy B. Cirrhosis C. AIDS D. Isonatremic dehydration

A. Salt-losing tubulopathy

Which gene has been implicated most frequently in the development of cancer? A. Tp53 gene B. Rb gene C. T21 gene D. None of these is implicated in the development of cancer

A. Tp53

Coup/countercoup occurs due to which type of injury mechanism? A. Traumatic injury B. Pressure injury C. Excitation injury D. Ischemic injury

A. Traumatic injury

Your grandfather has colon cancer but then it migrates to the liver. The affinity for movement to another specific organ is called: A. Tropism B. Migratism C. Magnetism D. Malignancy

A. Tropism

Which of the following terms indicates the dynamic steady state that the body strives to achieve every day? A. Homeostasis B. Mortality C. Morbidity D. Health

A.Homeostatsis

Your grandmother is diagnosed with congestive heart failure and is told that she has 6 months to live. This prediction is referred to as her: A. Prognosis B. Diagnosis C. Morbidity rate D. Prevalence

A.Prognosis

The nurse evaluates a client for manifestations of a disease. Which assessment data are considered signs? Select all that apply. A. Dizziness B. Dilated pupils C. Tachycardia D. Itching E. Skin rash F. Pain

B,C,E

What is the normal tidal volume for a person at rest? A. 3000 mL B. 500 mL C. 4600 mL D. 1100 ml

B. 500 mL

An 81-year-old female client in a subacute medical unit of a hospital has developed an oral Candida albicans infection. Which phenomena would the client's nurse suspect as a key contributing factor to her infection? A. The airborne communicability of yeast and molds and subsequent inhalation B. Antibiotic therapy that eliminated normal bacterial flora C. The ability of fungi to remain latent until the host reaches an immunocompromised state D. The moist and temperature-suited oral environment of the client's mouth

B. Antibiotic therapy that eliminated normal bacterial flora

Treatment of chronic conditions associated with cerebral atrophy A. Are effective at restoring neuronal function B. Are targeted toward slowing neuronal injury and atrophy C. Are best initiated when advanced signs and symptoms are evident D. Are not indicated

B. Are targeted toward slowing neuronal injury and atrophy

Which of the following major pathways occurs when a hormone is produced in a cell and can have an impact on neighboring cells and on its own cell? A. Paracrine pathway B. Autocrine pathway C. Synaptic pathway D. Neuroendocrine pathway

B. Autocrine pathway

Cell-mediated immunity is involved in resistance to infectious diseases caused by bacteria and some viruses. It is also involved in cell-mediated hypersensitivity reactions. Which of these does not cause a cell-mediated hypersensitivity reaction? A. Poison ivy B. Blood transfusion C. X-ray dye D. Latex

B. Blood transfusion

Which area of the brain is responsible for respiration? A. Frontal lobe B. Brain stem C. Diencephalon D. Midbrain

B. Brain stem

A nurse caring for a client with progressive cancer notes that the client has experienced significant loss of skeletal muscle and fat. The nurse documents this as: A. Waning syndrome B. Cachexia C. Anemia D. Anorexia

B. Cachexia

Which of the following cells is least likely to develop into a neoplasia? A. Epithelial cell B. Cardiac myocyte C. Lymphocyte D. Hepatocyte

B. Cardiac myocyte

Which is the most significant difference between acute lymphocytic and acute myelogenous leukemia? A. ALL affects adults and AML affects children most often B. Cell type affected C. Diagnostic tests used D. Clinical manifestations

B. Cell type affected

You have admitted a 20-year-old male to the emergency room with a history of asthma. He is having an acute asthma attack and is wheezing, fighting for air, hypoxic, and afraid. What is causing these acute symptoms? A. Relaxation of bronchial smooth muscle with dry mucous membranes B. Constriction of the bronchial smooth muscle and air trapping C. Acute destruction of lung tissue D. Contraction of the elastic fibers of the lung

B. Constriction of the bronchial smooth muscle and air trapping

A client with primary lung disease has developed right heart failure. The health care provider would document this as: A. Adult respiratory distress syndrome B. Cor pulmonale C. Cardiac tamponade D. Primary hypertension

B. Cor pulmonale

Excess cortisol is represented by which condition? A. Addison disease B. Cushing syndrome C. Diabetes insipidus D. Hyperthyroidism

B. Cushing syndrome

An epidemiologist is working in a rural community that has experienced a sudden and alarming increase in the incidence of HIV/AIDS. What is the main focus of epidemiology in this context? A. Developing innovative treatments for HIV B. Examining patterns in the way that HIV is spreading C. Performing pharmacologic testing of antiretroviral drugs D. Exploring the biochemical characteristics of the HIV virus

B. Examining patterns in the way HIV is spreading

The nurse is reviewing assessment documentation of a client's wound and notes "purulent drainage." The nurse would interpret this as: A. Exudate that is watery fluid, low in protein B. Exudate containing white blood cells, protein, and tissue debris C. Exudate that resulted from leakage of red cells D. Exudate containing large amounts of fibrinogen

B. Exudate containing white blood cells, protein, and tissue debris

Joe has many risk factors for the development of lung cancer. Which of these is NOT modifiable? A. Smoking B. Family history of lung cancer C. Working in an asbestos-filled area D. Poor nutritional intake

B. Family history of lung cancer

Your neighbor again comes to your door (see previous question). She has been running in the snow and it is cold outside. She has a headache and her heart is pounding. Again you check her blood pressure and find it to be HIGH. What could you do this time right in your home to decrease her blood pressure? A. Have her drink some hot chocolate B. Have her lay down on your couch C. Let her take a hot shower D. Give her something salty to eat

B. Have her lay down on your couch

The nurse is caring for a client with acute primary respiratory acidosis. When determining the cause of the acidosis the nurse is aware that which imbalance is most common? A. Decreased CO2 retention B. Impaired alveolar ventilation C. Increased metabolic acids D. Renal bicarbonate retention

B. Impaired alveolar ventilation

Which physiologic response increases an obese woman's risk for the development of breast cancer? A. Exaggerated triglyceride formation B. Increased sex hormone production C. Insulin resistance D. Chronic inflammation

B. Increased sex hormone production

Fluid loss in response to hypervolemia is promoted by: A. Stimulating secretion of ADH, promoting urinary sodium and water elimination B. Inhibiting the secretion of aldosterone, promoting urinary sodium and water elimination C. Lowering mean arterial pressure D. Administering osmotically active fluids

B. Inhibiting the secretion of aldosterone, promoting urinary sodium and water elimination

If a patient gets air in his or her pleural space, this results in: A. Pneumonia B. Pneumothorax C. Pleuritis D. Plethora

B. Pneumothorax

A patient is asked to collect a 24-hour urine test to check a hormone level. Why is the 24-hour urine needed? A. To measure female reproductive hormone levels B. To obtain a measurement of hormone secretion over time C. It is easier than obtaining a blood sample D. Urine is not an effective method of measuring hormone levels

B. To obtain a measurement of hormone secretion over time

An older adult client asks the nurse why so many older people develop heart failure. The bestresponse would be increased: A. Myocardial metabolism B. Vascular stiffness C. Response to beta-adrenergic stimulation D. Cardiac tone and compliance

B. Vascular stiffness

A client asks the nurse how a broad-spectrum antibiotic works. The best response would be that they are active against: A. Only Gram-positive infections. B. Only gram-negative infections. C. A wide variety of Gram-positive and Gram-negative bacteria. D. A wide variety of viruses.

C. A wide variety of Gram-positive and Gram-negative bacteria

Hyperlactatemia due to drug treatment is a complication in which of the following conditions? A. Salt-losing tubulopathy B. Cirrhosis C. AIDS D. Isonatremic dehydration

C. AIDS

A client is brought to the emergency department reporting shortness of breath. Assessment reveals a full, bounding pulse; severe edema; and audible crackles in lower lung fields bilaterally. Which action by the nurse is most appropriate? A. Administer hypertonic IV solution as ordered. B. Administer IV solution containing potassium as ordered. C. Administer diuretics as ordered. D. Arrange for a STAT electrocardiogram.

C. Administer diuretics as ordered

Which is the most common cause of atelectasis? A. Inflammation of the pleura B. Chronic, forceful coughing C. Airway obstruction D. Pulmonary embolism

C. Airway obstruction

The molecules that are recognized as foreign on allografts are called: A. MHC proteins B. Antigens C. Alloantigens D. Autoantigens

C. Alloantigens

An example of anion exchange includes: A. Sodium and hydrogen exchange B. Sodium and chloride exchange C. Bicarbonate and chloride exchange D. Hydrogen and bicarbonate exchange

C. Bicarbonate and chloride exchange

Which of these is found in the cell wall of gram-negative bacteria? A. Exotoxin B. DNA C. Endotoxin D. Prion

C. Endotoxin

After several months on a waiting list, a 44-year-old male received a liver transplant 5 days ago. In the last 36 hours, he has developed a rash beginning on his palms and soles, along with abdominal pain and nausea. It has been determined by his care team that the immune response that is causing his symptoms originates not with his own compromised immune components but with those introduced with his new organ. This man's most likely medical diagnosis is: A. Hyperacute organ rejection B. T-cell mediated graft rejection C. Graft-versus-host-disease (GVHD) D. Acute transplant rejection

C. Graft vs. host defense (GVHD)

Which of the following is an important characteristic of M. tuberculosis (the microbe that causes TB) and helps to explain its pathogenesis? A. It is a large and fast-growing microbe B. It produces toxins and these toxins destroy lung tissue C. It is resistant to destruction and can sit dormant for years D. It cannot be detected or diagnosed until the disease is well advanced.

C. It is resistant to destruction and can sit dormant for years

While the nurse is performing a skin assessment on a dark-skinned client, the nurse notes that the client has a healed wound on the leg but that the wound has an excess of scar tissue. The nurse documents this as: A. Remodeling. B. Epithelialization. C. Keloid. D. Proud flesh.

C. Keloid

The nurse is evaluating the bloodwork results of a client with an infected leg ulcer. The white blood cell count is 18,000 cells/uL. The nurse interprets this as: A. Lymphadenitis B. Neutropenia C. Leukocytosis D. Lymphocytosis

C. Leukocytosis

Parkinson disease is characterized by A. Neurofibrillary tangles B. Amyloid plaque C. Lewy bodies D. Tau

C. Lewy bodies

When caring for the client with portal hypertension and ascites, which dietary intervention does the nurse suggest to prevent the progression of fluid accumulation? A. Reduce protein intake. B. Avoid dairy products. C. Limit intake of sodium. D. Consume foods high in potassium.

C. Limit intake of sodium

Which of the following markers identifies a nucleated body cell? A. CD4 B. BCR C. MHC 1 D. MHC 2

C. MHC 1

The nurse is caring for a client with worsening respiratory acidosis. Which of these interventions does the nurse anticipate if the client's condition continues to deteriorate? A. Introduction of large muscle exercise B. Reducing the amount of supplemental oxygen C. Mechanical ventilation D. Resolution of the underlying emotional cause

C. Mechanical ventilation

A client has developed hydrocephalus and asks the nurse what may have caused this to occur. The best response would be: A. Excessive CSF flow in the ventricular system B. Deficient cerebrospinal fluid C. Overproduction of cerebrospinal fluid D. Increased reabsorption of cerebrospinal fluid

C. Overproduction of cerebrospinal fluid

Which statement best describes a secondary disorder of endocrine function? A. The disorder occurs from hypothalamic dysfunction resulting in understimulation of the target organ. B. The disorder occurs when the target organ becomes dysfunctional. C. The disorder occurs when the target organ is normal, but stimulating hormones alter its function. D. The disorder occurs in the target gland responsible for producing the hormone.

C. The disorder occurs when the target organ is normal, but stimulating hormones alter its function.

The nurse is teaching a client who has been newly diagnosed with hypothyroidism about the function of the thyroid. Which statement about the role of the thyroid gland is most accurate? A. The thyroid gland is responsible for regulating serum calcium levels. B. The thyroid gland promotes development of secondary sex characteristics. C. The thyroid gland is responsible for increasing the metabolic rate. D. The thyroid gland releases neurotransmitters when the "flight or flight" mechanism is stimulated.

C. The thyroid gland is responsible for increasing the metabolic rate.

A client has sustained a full-thickness burn involving subcutaneous tissue, muscle, and bone. Which term will the nurse use when documenting this type of burn? A. Second- and third-degree burns B. First- and second-degree burns C. Third-degree burn D. Second-degree burn

C. Third degree burns

A hypersensitivity reaction resulting from complement activation due to insoluble antigen-antibody deposition is an example of: A. Type 1, immediate hypersensitivity reaction B. Type 2, antibody-mediated reaction C. Type 3, immune complex reaction D. Type 4, cell-mediated reaction

C. Type 3, immune complex reaction

Congenital heart defects can cause a right heart-to-left heart shunting of blood that results in increased: A. Right ventricle workload B. Right atrial blood volume C. Unoxygenated blood flow D. Pulmonary blood volume

C. Unoxygenated blood flow

An older adult client newly diagnosed with systolic hypertension asks her health care provider why this happens. Which response is most accurate? A. "Everyone over the age of 50 tends to have their blood pressure creep up over the years." B. "If you slow down and rest more, your blood pressure will more than likely return to its normal level." C. "With age, your arteries lose their elasticity and are replaced with collagen, which makes your arteries stiffer." D. "Your heart has to work harder to pump blood through your vessels as you get older."

C. With age, your arteries lose their elasticity and are replaced with collagen, which makes your arteries stiffer.

Which of these is a single-celled organism that is about the size of a red blood cell and reproduces by a budding process? A. Virus B. Mold C. Yeast D. Bacteria

C. Yeast

Which type of neuron synthesizes and secretes norepinephrine as the primary neurotransmitter? A. Serotonergic B. Cholinergic C. Nicotinic D. Adrenergic

D. Adrenergic

The nurse teaches the client with hyperparathyroidism that he is at risk for: A. Increasing serum magnesium. B. Rickets. C. Increasing phosphate excretion. D. Developing kidney stones.

D. Developing kidney disease

The nurse is caring for a client with ketoacidosis who is complaining of increasing lethargy and occasional confusion following several weeks of rigid adherence to a carbohydrate-free diet. The nurse understands which phenomenon is most likely occurring? A. Metabolism of dietary fats without the buffer action of carbohydrates results in the catabolism of ketoacids. B. Decreased carbohydrate intake induces insulin deficiency and consequent ketoacidosis. C. High fat, low carbohydrate dietary intake is associated with respiratory acidosis. D. In the absence of carbohydrate energy sources, her body is metabolizing fat and releasing ketoacids.

D. In the absence of carbohydrate energy sources, her body is metabolizing fat and releasing ketoacids.

Which compartment contains the greatest amount of body water? A. Transcellular B. Plasma C. Interstitial D. Intracellular

D. Intracellular

A young, healthy adult helps lift a sofa to move it 25 ft (7.5 m). The nurse knows that which source of energy will the client likely use in this process? A. Glucose B. Amino acids C. Fatty acids D. Stored adenosine triphosphate (ATP)

D. Stored adenosine triphosphate (ATP)

Which lobe of the brain performs functions of perception, long-term memory, and recognition of auditory stimuli? A. Occipital lobe B. Frontal lobe C. Parietal lobe D. Temporal lobe

D. Temporal lobe

A client has been diagnosed with a cerebral aneurysm and placed under close observation before treatment commences. Which pathophysiologic condition has contributed to this client's diagnosis? A. Deficits in the autonomic control of blood pressure B. Increased levels of cerebrospinal fluid C. Impaired synthesis of clotting factors D. Weakness in the muscular wall of an artery

D. Weakness in the muscular wall of an artery


Conjuntos de estudio relacionados

Adult Health II Quiz 2: Heart Failure / Infectious & Inflammatory Cardiac Disorders

View Set

Ch. 8: Commercial Property Insurance

View Set

AP Euro Chapter 16 correct answers

View Set

Steps In Pre-Service Procedure, Basic Table Setup

View Set

Chapter 4 Carbon and the Molecular Diversity of Life

View Set

SOCW 2030 Interviewing Techniques Exam

View Set

Risk factors can be categorized as congenital conditions (present at birth) or acquired defects (occurring after birth). Question 4 0 / 0 pts What is the difference between morphology and histology? Your Answer: Histology- the changes in the cell, this

View Set